You are on page 1of 62

1° EXAMEN TIPO ENAM A.

Probable dengue
B.Probable dengue grave
C.Probable dengue hemorrágico
D.Dengue confirmado
NOMBRE………………………………………………………….
E.Dengue descartado
SECCION …………………………………………………………

Infectología
7. : La gingivoestomatitis herpética es la forma clínica
1. Paciente de 40 años de edad, que viene más frecuente de la infección primaria debida a :
directamente desde Abancay al servicio de A. Herpes virus hominis tipo 1
Emergencia, en donde se evidencia paciente B. Herpes virus hominis tipo 2
soporoso, con fiebre de 40°C. Severa palidez de C. Herpes virus hominis tipo 6
piel y mucosas, subictérico. Hepatoesplenomegalia. D. Herpes virus hominis tipo 7
Análisis: hemoglobina: 4 g%; leucocitos:18000, E. Herpes virus hominis tipo 8
reticulocitos:15%; test de Coombs: negativo; test de
ham:negativo; gota gruesa:negativo. La posibilidad 8. : ¿Cuál es el agente etiológico de la colitis
diagnóstica es: pseudomembranosa? :
A. Infección por Clostridium perfringes A. Clostridium perfringes
B. Anemia hemolítica autoinmune B. Clostridium botulinum
C. Hemoglobinuria paroxística nocturna C. Clostridium difficcile
D. Malaria falciparum D. Clostridium tetani
E. Bartonelosis E. Clostridium novyi

2. El antibiótico de elección en una mujer de 32 años, NEUROLOGIA


con cervicitis mucopurulenta y PCR positivo para 9. Al evaluar un paciente politraumatizado se
Chlamydia trachomatis es: encuentra que abre los ojos y retira la mano ante
A. Doxicilina estímulos dolorosos. Emite sonidos
B. Azitromizina incomprensibles. Le corresponde un puntaje en la
C. Penicilina benzatínica escala de Glasgow de:
D. Ciprofloxacina a. 3 b. 4 c. 6
E. Metronidazo d. 8 e. 12

10. Paciente de 72 años, con antecedente de


hipertensión arterial. Presenta disminución de
3. Mujer de 35 años, procedente de Cajamarca, fuerza muscular en hemicuerpo izquierdo, a
presenta desde hace 3 meses dolor lumbar, fiebre y predominio de miembro inferior proximal, con
sudoración parcelar a predominio nocturno. Al relajación de esfínteres. El diagnóstico más
examen FC: 100 x minuto FR: 20 x minuto T: probable es:
38.5ºC, pálida con adenopatías generalizadas, a. Isquemia en territorio de la arteria cerebral
hepato y esplenomegalia. ¿Cuál es el diagnostico anterior derecha
probable? b. Isquemia en territorio vertebro basilar
A.Fiebretifoidea c. Infarto carotídeo
B.Tuberculosis de columna vertebral d. Infarto basilar
C.Fiebre malta e. Infarto de la arteria cerebral media
D.Malaria .
E. Leptospirosis
11. Paciente mujer de 18 años que hace 2 semanas
4. Varón de 57 años, antecedente de alcoholismo. presentó un cuadro diarreico que se autolimitó.
Hace 2 semanas fue diagnósticado de tuberculosis Acude a consulta por presentar debilidad muscular
pulmonar con BK en esputo positivo. Antes de y parestesia. Al examen físico se encuentra
iniciar el tratamiento con isoniacida, rifampicina, disminución de fuerza muscular simétrica de forma
etambutol y pirazinamida ascendente y arreflexia. ¿Cuál es el mejor
A. BK en orina y en jugo gástrico diagnóstico?
B. Bilirrubinas y transaminasas a. Miastenia Gravis.
C. Hemograma y creatinina b. Botulismo.
D. Dosaje de gases arteriales c. Sindrome Guillain Barré.
E. Sodio y potasio d. Esclerosis lateral amiotrófica.
e. Esclerosis múltiple.
5. El vector transmisor de la fiebre amarilla urbana es:
A) Anopheles 12. En un paciente con hiperreflexia osteotendinosa,
B) Lutzomya verrucarum ¿qué neurona está lesionada?
C) Aedes aegypti a. Talámica
D) Haemagogus spegazzinil b. Motora medular
E) Triatoma infestans c. Sensitiva
d. Intercalar
6. Fiebre de menos de 5 días de duración, sin e. Motora cortical
afección de las vías aéreas superiores con cefalea
y mioartalgias. ¿Cuál es el diagnóstico más 13. Varón de 65 años de edad que presenta un cuadro
probable? de hemiparesia braquicorural izquierda y en el
transcurso de 12 horas recupera la movilidad. d. Maníaco depresivo
Probablemente se trata de: e. Fóbico
a. Tumor en región temporal de hemisferio derecho
b. Hemorragia intracraneana del hemisferio 19. -Mujer de 25 años, estudiante universitaria, madre
izquierdo consulta porque nota cambios de conducta,
c. Ataque isquémico transitorio del hemisferio algunas veces se torna irritable, aislada y poco
derecho comunicativa. En la última semana presenta
d. Cisticercosis cerebral insomnio y tendencia al llanto, ha bajado en su
e. Tromboembolia de la carótida derecha rendimiento académico. ¿Cuál es el diagnóstico
más probable? PSIQUIATRIA
a. Ansiedad
b. Neurosis
14. Varón de 30 años de edad, que presenta
c. Depresión
convulsiones en el servicio de Emergencia. ¿Cuál
d. Manía
de los siguientes fármacos administraría?:
e. Distimia
a. Valproato
b. Metilfenitoína
c. Difenilhidantoína
20. Varón soltero de conducta de aislamiento social,
d. Carbamazepina
negligencia en su aseo personal, de 21 años de
e. Diazepam
edad, quien desde hace 8 meses presenta voces
que lo insultan, refiere que la policía lo persigue y
15. Niño de 2 meses de edad, es llevado a Emergencia
que es controlado por la antena parabólica de su
por presentar convulsiones tónico-clónicas
barrio. El diagnóstico más probable es:
generalizadas desde hace 45 minutos, sin
a. Trastorno delirante persistente
recuperar la conciencia. Antecedente de asfixia al
b. Psicosis reactiva breve
nacer. Presenta fiebre de 39 °C. ¿Cuál es el
c. Trastorno esquizoafectivo
diagnóstico más probable?:
d. Trastorno esquizofrenoide
a. Crisis convulsiva
e. Esquizofrenia
b. Convulsión febril compleja
c. Estatus convulsivo
21. Varón de 45 años de edad con conflictos
d. Convulsión febril simple
conyugales desde hace 2 años. Se queja de
e. Epilepsia
dificultad para conciliar el sueño, contractura
16. Paciente de 55 años de edad, que sufre una
muscular, dolor de nuca y espalda, irritabilidad,
caída desde tres metros de altura. Se diagnóstica
parestesias en miembros superiores, expectación
TEC con hipertensión intracraneana. Glasgow 7.
aprehensiva. El diagnóstico probable es:
¿Cuál es la conducta inmediata a seguir?:
a. Hipocondriasis
a. Intubación endotraqueal y ventilación asistida
b. Trastorno conversivo
b. Administración de manitol en bolo
c. Trastorno de adaptación
c. Reanimación con solución salina hipertónica
d. Dolor psicógeno
d. Aplicación de dexametanosa
e. Trastorno de ansiedad generalizada
e. Aplicación de hidrocortisona1
22. Tratamiento de elección del trastorno bipolar:
a. Sales de Litio
16. Mujer de 32 años, llega a consulta con cefalea
b. Carbamazepina
intensa, vómitos y rigidez de nuca. El LCR presenta
c. Sertralina
pleocitosis linfocitaria. ¿Cuál es el diagnóstico más
d. Haloperidol
probable?
a. Meningitis viral
23. Son factores mayores de riesgo para el suicidio:
b. Meningitis por hongos
a. Edad mayor de 65 años y aislamiento social
c. Meningitis bacteriana
b. Edad mayor de 65 años y abuso de alcohol
d. Meningitis por VIH
c. Enfermedad mediana crónica y género masculino
e. Hemorragia subaracnoidea
d. Abuso de alcohol y aislamiento social
e. Enfermedad mediana crónica y aislamiento
17. Dentro de los síntomas de la migraña NO se
social
considera:
a. Localización unilateral
24. ¿cuál de las siguientes características NO
b. Cefalea pulsátil
corresponde al síndrome neuroléptico maligno?
c. Aura asociada
a. Es un trastorno febril
d. Otalgia
b. Es consecuencia del uso de fármacos como el
e. Alivio de los síntomas al dormir
haloperidol
c. Produce hiperreflexia
PSIQUIATRIA
d. Produce infarto cerebral
e. Aumenta la creatin-fosfoquinasa (CPK).
18. Un paciente de 40 años con delusiones auditivas y
de persecución, desconfiado y celoso, es un
paciente:
a. Histérico
b. Paranoico
c. Obsesivo compulsivo
DERMATOLOGIA

25. Es un proceso inflamatorio crónico que se asocia 32. Mujer de 40 años de edad, presenta
con la presencia de pelos en el tejido graso predominantemente en la región nasogeniana
subcutáneo: legiones papulosas eritomatosas y descamaticas,
a. Hidradenitis supurada. las cuales se exacerban con alguna comida y
b. Celulitis. tensión emocional. el diagnostico mas probable es:
c. Absceso a. Piodermititis piógena
d. Piodemitis. b. Pénfigo vulgar.
e. Quiste pilonidal c. Rosácea
d. Dermatitis alérgica.
26. Mujer de 20 años de edad, peladora de espárragos e. Psoriasis
desde hace 4 meses, presenta lesiones
eritematosas pruriginosas micropapulares
localizada en dorso de antebrazos, cara y cuello. 33. Varón de 27 años presenta lesiones dérmicas
Las lesiones también se han identificado en maculo eritematosas vesiculares, descamativas y
algunas compañeras de trabajo. No presenta bordes escamosos sobreelevados en cara, tronco y
antecedentes de atopía. Su primera impresión extremidades que se extienden en forma centrifuga.
diagnóstica es: ¿Qué tipo de dermatofitosis es?
a. Acarosis a. Candidiasica cutánea
b. Dermatitis por contacto b. Tina circinada
c. Pitiriasis rosada c. Pitiriasis versicolor
d. Dermatofitosis d. Tina capitis
e. Rosácea e. Tiña corporis
27. ¿Cuál es la característica más importante de la
lesión para el pronóstico de un paciente con 34. ¿Cuál de las siguientes lesiones NO se considera
melanoma maligno? una lesión primaria de piel?
a. Grosor a. Pápula b. Mácula
b. Diámetro c. Cicatriz d. Roncha
c. Borde e. Pústula
d. Color
e. Vascularidad 35. La lesión elemental clásica de la urticaria es:
a. Comedón b. Ampolla
28. Surfista de 30 años de edad presenta una lesión c. Vesícula d. Mácula
verrucosa en el labio inferior, única, con e. Habón.
eritroplaquia en el paladar ¿Cuál es el diagnóstico NEFROLOGIA.
más probable?
a. Queratosis actínica 36. Paciente con cuadro de deshidratación severa, que
b. Carcinoma basocelular presenta un volumen urinario de 10 mL/hora,
c. Carcinoma espinocelular creatinina sérica 2 mg/dL, osmolaridad urinaria
d. Queratosis seborreica 1.025, sodio urinario 10 mEq/L. El diagnóstico más
e. Melanoma maligno probable es insuficiencia:
A. Renal por papilitis necrótica
29. Paciente que presenta en el cuero cabelludo costra B. Renal con necrosis tubular
láctea no pruriginosa. ¿Cuál es el diagnóstico más C. Post-renal
probable? D. Pre—renal
a. Pitiriasis rosada E. Renal crónica
b. Eritema polimorfo
c. Dermatitis seborreica 37. ¿Cuál de los parámetros señalados caracteriza
d. Psoriasis mejor a la insuficiencia renal aguda prerenal?:
e. Liquen plano A. Relación urea/nitrógeno < 3
30. Mujer de 32 años, ama de casa, presenta eritema B. Excreción fraccionada de Na > 1
pruriginoso, lesiones descamativas en las manos. C. Sodio urinario > 20 mEq/L
Antecedente de asma bronquial y tuberculosis D. Sodio urinario < 20 mEq/L
pulmonar en la infancia ¿Cuál es la presunción E. FENa < 1%
diagnóstica?
a. Dermatitis seborreica 38. Paciente diabética hospitalizada por infección
b. Tuberculosis cutánea urinaria de 4 días de evolución ¿Cuál es el agente
c. Dermatitis atópica etiológico más probable?
d. Dermatitis de contacto A. Klebsiella sp.
e. Psoriasis B. Proteus mirabilis
C. E. coli
31. Lesión descamativa plateada, sobre una mancha D. Pseudomona aeruginosa
roja. ¿Cuál es el diagnóstico más probable. E. Estafilococo saprophyticus
a. Dermatitis seborreica
b. Liquen simple crónico 39. En el dosaje cuantitativo de proteinuria de 24 h
c. Psoriasis ¿Cuál es el punto de corte para considerar
d. Tiña corporis síndrome nefrótico?
e. Liquen plano A. Mayor de 5 gr
B. Mayor de 7 gr E. Aumento del VEF1 y disminución del VR
C. Entre 1 y 2 gr
D. Mayor de 3.5 gr 48. ¿Cuál es el tratamiento antimicrobiano de elección
E. Mayor de 6 gr en caso de neumonía adquirida en la comunidad?:
A. Tetraciclinas más penicilinas
40. : ¿Qué signos semiológicos permiten determinar B. Cefalosporina de tercera generación más
hipocalcemia? macrólidos
A. Chvostek y Lahey C. Aminoglucósidos más Quinolonas
B. Lahey y Túnel D. Sulfas mas carbapenem
C. Trousseau y Lahey E. Lincosaminas más macrólidos
D. Tunel y Chvostek
E. Chvostek y Trousseau 49. l esquema de elección para el tratamiento de la
tuberculosis pulmonar en el adulto no tratado
41. Madre con TBC pulmonar acude con su hijo previamente es:
hipotónico e hipoactivo. Refiere que solo lo A. 2HRSE/4R2H2
alimenta con agua, infusiones y sopas, por falta de B. 2RHRZSE/4R2H2E2
dinero. ¿Cuál es el trastorno hidroelectrolítico que C. 2HRZ/4R2H2
presenta? D. 2HRZE/4R2H2
A. Hiperfosfatemia E. 2HRZE/7R2H
B. Hipernatremia
C. Hiponatremia
D. Hiperkalemia 50. La dosis diaria de lsoniazida utilizada con mayor
E. Hipokalemia frecuencia en el adulto con tuberculosis, es:
a) 5mg/kg de peso
42. Varón de 92 años, acude a EMG por episodio b) 10 mg/kg de peso.
convulsivo. Examen físico: hemodinamicamente c) 15 mg/kg de peso.
estable, no edemas. Laboratorio. Sodio sérico en d) 20 mg/kg de peso.
108 mEq/L. En el tratamiento, la reposición de e) 25 mg/kg de peso.
sodio con solución salina es al :
A. 5% B. 3% C. 9%
D. 10% E. 15%
51. El complejo de Ghon es característico de:
a. Actinomicosis
43. Paciente de 14 años de edad, acude al Consultorio
b. LUES congénita
por presentar albuminuria masiva. Teniendo en
c. Tuberculosis secundaria
cuenta la histología del glomérulo, ¿Qué estructura
d. Tuberculosis primaria
se encuentra dañada?:
e. Nocardiosis
A. Arteriola eferente
B. Mácula densa CARDIOLOGIA
C. Células mesangiales
D. Podocitos 52. En el ciclo cardiaco la fase 0 se corresponde con:
E. Lámina basal a. Contracción isovolumetrica
NEUMOLOGIA. b. Relajación isovolumetrica
c. Eyección rápida
44. En el asma aguda, el fármaco utilizado en el d. Eyección lenta
tratamiento inicial es: e. Llenado
A. Metilxantinas
B. Corticosteroides 53. La ley de Starling explica bien los fenómenos
C. Beta- 2 agonistas de acción corta cardíacos de:
D. Beta- 2 agonistas de acción larga a.Postcarga.
E. Anticolinérgicos b.Precarga.
c. Frecuencia.
45. En adultos. ¿cuál es el valor normal de volumen de d.Conducción.
aire corriente en ml, obtenido en la espirometría? e.Excitación
A. 600 B. 400 C. 300
D. 500 E. 200 54. La proteína que almacena el hierrointracelular es:
a. Rubredoxina
46. Tratamiento de rescate de crisis asmática: b. Hemoglobina
A. Anticolinérgicos c. Ferritina
B. Teofilina d. Transferrina
C. B agonistas e. Superóxido dismutasa.
D. Hidrocortisona
E. Prednisona 55. La heparina sigue siendo el anticoagulante de
elección en muchos ámbitos clínicos. Las dosis de
47. En un paciente con EPOC, la prueba de función la heparina se deben ajustar de acuerdo a
pulmonar muestra: determinacionesrepetidas de:
A. Disminución de CVF y VEF1 normal a. Los tiempos de protrombina.
B. Disminución del VEF1 y aumento de CVF b.Los tiempos de coagulación.
C. Disminución del VEF1 y CVF c. Los tiempos de cefalina.
D. Aumento del VEF1 y CVF d.Los tiempos de tromboplastina parcial activada.
e.Del factor VIII de la coagulación. b. Estimular la glucogenólisis
c. Estimular la liberación de insulina
56. Los efectos antiplaquetarios del ácidoacetilsalicílico d. Estimular la lipolisis
persistenpor: e. Estimula la cetogenesis
a. 4-6 días.
b. 1-2 días. 63. Principal estimulo para la secreción de insulina:
c. 8-10 días. a.Incretinas b.GLP c. Lisina
d. 12-14 días. d.Arginina e.Glucosa
e. 16-18 días
57. Varón de 71 años, hipertenso, acude por
precordalgia intensa, EKG: supradesnivel 64. Paciente varón de 45 años con peso del 100 kg ,
del segmento ST en D II, D III, aVF e infradesnivel talla 1,55 cm. Consulta por polidipsia, poliurea ,
ST en V1, ¿Cuál es el diagnóstico más probable? polifagia y sed excesiva . se le saca los siguientes
exámenes de laboratorio.
a. IMA STE inferior. Glucosa en ayunas ayer: 120
b. IMA STNE posterior Glucosa ayunas hoy : 114
c. IMA STE posterior A que examen pasaría dicho paciente:
d. IMA STE inferoposterior a. Repetir otra glucosa en ayunas
e. IMA cara anterior b. Sacar glicemia postpandrial
c. Pedir hemoglobina glicosilada
58. Mujer de 65 años, con dx de falla cardiaca, desde d. Pedir Test tolerando oral de glucosa con
hace 3 meses presenta disnea y dolor precordial a glucosa anhidrida 75mg
pequeños esfuerzos por lo que permanece en
e. No pedir ningún examen.
reposo. Examen físico: en ortopnea, FC: 100X, FR
24X, ingurgitación yugular (++), y edema de 65. Cuál es el tumor más frecuente de tiroides:
miembros inferiores. ¿Cuál es la clase funcional A. Papilar
según NYHA? B. Folicular
A. III B. IV C. I C. Anaplasico
D. II E. V D. Medular
E. Metastásico
59. Varón de 56 años, con diagnóstico de Wolff
Parkinson White, presenta bruscamente taquicardia 66. Mujer de 35 años, acude a emergencia por
con pulso irregular de 180 latidos por minuto con presentar palpitaciones, cambios de conducta y
compromiso hemodinámico. ¿Cuál es la medida disnea. Al examen: presenta fascies ansiosa,
terapéutica más adecuada? sudoración y piel caliente. PA: 130/80 mmHg. FC:
A. Lidocaína 130 x min, FR: 28 x min. Cuello: tiroides 2.5 N,
B. Quinidina difuso y no doloroso. Aparato cardiovascular:
C. Cardioversión electrica taquicardia. ¿Cuál es el examen de laboratorio que
D. Amiodarona ayudaría a confirmar el diagnóstico?
E. Betabloqueador A. TSH, T3 y T4 libre
B. Creatinin fosfocinasa -MB
60. Un paciente con cardiopatía hipertensiva e C. Troponina T
isquémica acude a Emergencia por angina D. Dosaje de gases arteriales
moderada. El examen muestra un paciente pálido y E. Dímero D
diaforético. Su FC es de 150 por minuto y su PA
95/50 mmHg. El electrocardiograma muestra
67. Anciana de 70 años sin antecedente de DM es
complejos QRS de 0,14´´ e intervalos R-R iguales.
llevada a emergencia por presentar estado mental
Las ondas P se observan en forma inconstante, sin
guardar relación con los complejos QRS. El manejo alterado, cuadro de deshidratación severa, el
de primera línea en este paciente debe ser: familiar solo informa que días antes presentó
A. Infusión de amiodarona polidipsia, poliuria. El diagnóstico planteado seria:
B. Cardioversión eléctrica A. DM tipo 1
C. Verapamil EV B. DM tipo 2
D. Maniobras vagales C. Cetoacidosis diabética
E. Monitoreo y observación D. Coma hiperosmolar no Cetósico
E. Hipoglicemia
ENDOCRINOLOGIA.
68. 5 Las necesidades de insulina en un paciente
61. El paciente diabético tiene hiperglicemia en ayunas diabético disminuyen con: }
debido a la gluconeogénesis hepática, mencione el A. Cirugía
antidiabético que bloqueo este proceso metabólico B. Infecciones
y la reacción adversa más severa. C. Stress emocional
a.Glibenclamida – hipoglicemia D. Obesidad
b.Rosiglitazona – edemas E. Ejercicio
c. Metformina – acidosis láctica
d.Exenatide – pancreatitis 69. Son intolerantes a la glucosa:
e.Sitagliptina – Nefrotoxicidad A.Glicemia basal de 115
B.Glicemia a los 120 minutos de TTOG entre 140 y
200.
62. . No es efecto del glucagón C.Basal de 140
a. Estimular la gluconeogénesis
D.Glicemia a los 120 minutos de TTOG entre 200 y 77. La causa más frecuente de hemorragia digestiva
202. baja es:
E. Curva de glicemia patológica tras estimulo con A. Diverticulosis
GC B. Malformación arteriovenosa
C. Hemorroides
70. Paciente varón de 45 años, con glicemias de 118 y D. Angiodisplasia
135 mg/dl en 2 oportunidades diferentes. ¿Cuál es E. Ectasia vas
la conducta a seguir?:
A. Dar NPH. 78. En la pancreatitis aguda del adulto, ¿cuál de los
B. Hipoglicemiantes orales. siguientes criterios no corresponde a los de
C. iniciar dieta. Ranson?
D. Test de tolerancia a la glucosa. A. Leucocitosis >16000/𝑚𝑚3
E. Hb glicosilada + test de tolerancia a la glucosa. B. Hiperglicemia mas de 200
C. EDAD menos a 55 años
71. El reflujo gastroesofágico crónico puede producir: D. LDH en suero >400 UI/L
A. Candidiasis esofágica E. AST en suero >250 UI/L
B. Hernia hiatal
C. Infección por Helicobacter pylori de la mucosa 79. La zona del intestino que se afecta con más
gástrica frecuencia en la enfermedad de crohn es:
D. Divertículos esfágicos A. Recto
E. Esófago de Barret B. Colon derecho
C. Duodeno
72. Varón de 50 años de edad, alcohólico crónico; D. Yeyuno
presenta hematemesis masiva y melena. Pulso: E. Ileon terminal
120 por minuto, PA: 80/60 mmHg, palidez marcada,
abdomen batraciano, circulación colateral. La causa 80. La principal complicación de la pancreatitis aguda
más probable del sangrado es: es:
A. Cáncer gástrico A. Diabetes mellitus
B. Várices esofágicas B. Pseudoquiste pancreático
C. Gastritis erosivas C. Desnutrición
D. Síndrome de Mallory-Weiss D. Ictericia
E. Úlcera gástrica E. Esteatorrea

73. : Un marcador precoz de la disminución de la DR ROBINSON LEON ZULOETA


función hepática es: DOCENTE TC USS
A. Trombocitopenia
B. Prolongación del tiempo parcial de
tromboplastina
C. Hipoproteinemia
D. Anemia macrocítica
E. Prolongación del tiempo de protrombina

74. La Enfermedad Diarreica Aguda caracterizada por


transporte activo de solutos a la luz intestinal, es la
diarrea:
A. Por invasión de la mucosa
B. Osmótica
C. Secretora
D. Por aumento de la motilidad
E. Por mala absorción

75. En el tratamiento del helicobacter pylori, es cierto:


A. Se utiliza solo inhibidor de la bomba de protones
B. No se utiliza más de un antibiótico
C. Se utilizan 2 antibióticos y un inhibidor de la
bomba de protones
D. Se utilizan 3 antibióticos
E. Se utilizan un inhibidor de la bomba de protones
y un antiácido

76. Aproximadamente el 90% de los pacientes con


hematozequia sangra por el:
A. Recto
B. Esófago
C. Estómago
D. Duodeno
E. Colon
1° EXAMEN DE MEDICINA DE URGENCIAS

NOMBRE…………………………………………………………………………..AULA………………………

1. Anote ud. las 10 causas de RCP REVERSIBLES ( 5H - 5 T ).

2. Son arritmias cardiacas desfibrilables…………………………………….……………………………………....................


3. Son arritmias cardiacas no desfibrilables………………………………………………………………………………………..
4. 2 condiciones básicas del RCP BASICO no considerar ………………………………y las …………………………….
Son mas prioritarias que las ……………………………………………………………………………………………………………
5. En el RCP BASICO se harán …….………..comprensiones cardiacas ……………….ventilaciones
6. En el RCP AVANZADO se harán ………….. comprensiones cardiacas …………….ventilaciones con …………….ó el
paciente ………………………………
7. El RCP AVANZADO se diferencia del básico en que al paciente si se examina el …………… en diferentes regiones
corporales cuales: ………………………………………………………………………………………………………………………………
8. En el diagnostico de ICC según Framighan es correcto afirmar .
a. Ortopnea
b. Disnea + edemas en mm ii
c. Cardiomegalia en rayos + edemas en abdomen
d. Disnea paroxística nocturna + edema en piernas y tos nocturna
e. IY + Frecuencia cardiaca mayor del 120
9. Varón de 81 años con antecedente de diabetes y hipertensión ingresa ala emergencia por disnea , tos
nocturna mas ortopnea , además refiere dolor toraccico opresivo atípico sin diaforesis.
Examen físico: PA : 145/90 FC: 92 FR: 24 SO2: 90%
Pulmones: se ausculta crepitantes bibasales en ambos campos pulmonares .
Miembros inferiores; hay edemas +/ +++ mas frialdad distas +/ +++
EKG se observa el sgte patrón:

Problemas de salud …………………………………………………………………….


EKG: Ritmo………………………. FC…………………….. Eje eléctrico ………………………………..
QRS …………………………………. SOKOLOV…………….Diagnostico …………………………………

10. Patrón del cuadro clínico según KILLIB KIMBAL………………………………………………………..


11. Patrón hemodinámico ………………………………………………………………………………………………
12. Mujer de 89 años de edad con antecedentes de diabetes mellitus tipo II, hipertensión y dislipemia. Acude a
la consulta acompañada de su hijo por disnea que empeora progresivamente en el último mes.
En la última semana asocia edemas de miembros inferiores. Niega dolor torácico u otros síntomas
asociados. En la exploración física destacan, además de los edemas en miembros inferiores, leves
crepitantes en bases pulmonares, al llegar ala emergencia llega diaforesis mas vomito 2 veces con dolor
abdominal en epigastrio.

EKG nos muestra:

Problemas de salud, Prioridad………………………………………………………………………………………………….

Síndromes ………………………………………………………………………………………………………………………………..

Diagnostico ……………………………………………………………………………………………………………………………….

13. Ekg muestra: RITMO ………………………………………….FC………………………………………………


Eje electrico ……………………………………………………… QRS …………………………………………..
Además se observa ………………………………………………………………………………………………….
Diagnostico EKG ……………………………………………………………………………………………………..

14. Tratamiento que ud en URGENCIAS aplicaría……………………………………………………………


…………………………………………………………………………………………………………………………………………………………………………
…………………………………………………………………………………………………………………………………………………………………………
…………………………………………………………………………………………………………………………………………………………………………
…………………………………………………………………………………………………………………………………………………………………………

15. En el siguiente EKG es de un paciente diabético de 65 años de edad, que ingresa por dolor en epigastrio, mas
diarrea 4 deposiciones desde las 3 am de hoy día, ingresa alas 6am ala emergencia del hospital donde presento
2 vómitos de contenido alimentario de la cena, y los siguientes signos vitales.
PA: 130/80 pulso; 90 por min SO2 . 90 % . su EKG 6am informa lo siguiente:
RITMO : QRS :

FRECUENCIA CARDIACA: ST:

EJE ELECTRICO: DIAGNOSTICO EKG:

16. ADEMAS ANOTE UD. CUAL es el tratamiento que ud. Indicaría en la emergencia de este paciente:
…………………………………………………………………………………………………………………………………………………………………………
…………………………………………………………………………………………………………………………………………………………………………
…………………………………………………………………………………………………………………………………………………………………………
………………………………………………………………………………………………………………………………………………………………………

17. Paciente de 50 años obeso con ac de diabetes ortopnea nocturna ( duerme sentado ), al
mellitus ingresa ala emergencia por dolor de examen se ausculta crepitantes en los 4
pecho, taquicardia, con hipotensión 80/60, al cuadrantes pulmonares, en el cuello hay IY (
EKG presenta descenso del ST mas de 2 mm de +++ / +++ ), al palpar los pies se encuentran
voltaje en V1 a V5, cual de las siguientes frios y sudorosos , hoy hace 4 horas llega el
premisas es falsa en cuanto al tratamiento. paciente algo confuso : Glasgow: 10 puntos.
a. Debe recibir 2 antiagregacion plaquetaria En las ultimas 4 horas diuresis : 100 cc , PA:
minimo 2 años con ASA´+ clopidrogel 80/60 FC: 100 por minuto SO2. 85%
b. Debe recibir Enoxaparina SC por 14 dias Que patrón hemodinámico se encuentra –
c. En la estratificación TIMI tiene 4 puntos tratamiento a instaurar.
por lo que este debe recibir tratamiento a. Caliente, seco – furosemida
medico b. Caliente – húmedo – furosemida
d. Además debe recibir Morfina para calmar c. Frio , húmedo - dobutamina +
el dolor de pecho noradrenalina
e. Debe recibir Oxigeno adicional además de d. Frio, seco - dopamina
Atorvastatina 80 mg una tableta por la e. Frio , seco - Nacl 9x 1000 cc.
noche. 19. Paciente mujer de 65 años que ingresa por
18. Paciente varón de 65 años de edad con disnea progresiva hace 14 días , esta va en
antecedente de hipertensión arterial mas un aumento con el paso de los días, se agrega 7
antecedente de infarto de miocardio ST días después edemas en mm ii ++ / +++
elevado con tratamiento después de las 12 bilateral , mas tos con expectoración
horas , hace 7 dias ingresa por falla cardiaca blanquecina en los últimos 3 días
descompensada presenta con disnea 1 día antes de ingreso se agrega ortopnea
progresiva nocturna por la noche, mas nocturna ( duerme con almohadas ) con disnea
al cepillarse los dientes o irse al baño, luce ……………………… alternativo …………………..…..
somnoliento. prioridad………………………
Al examen físico: a. Ceftriaxona - ciprofloxacino – II
PA: 160/90mmHg FC:115x’ FR:42x’ b. Ceftazidima - colistina - II
SatO2:80% (FIO:21%) c. Ciprofloxacino - Colistina – II
P: 72Kg T:165cm IMC:26.4 d. Meropenen – amoxicilinina – I
Torax: crepitantes en ambas bases bilateral e. Dicloxacilina - Bactrin forte - III
Piel: fría ++/+++ en dedos de manos y pies 24. Paciente que ingresa ala emergencia con tos ,
bilateral . mal estar general, PA; 100/60
Diagnostico – ( grado de Disnea –AHA ) – FC: 110 FR: 24. T°: 38 ° C. EX DE
patrón hemodinámico . LABORATORIO: leucocitos : 16000 / campo .
a. Edema agudo de pulmón - 3 – seco , frio su diagnostico:
b. ICC – 4 - húmedo – caliente a. Infección
c. Edema agudo de pulmón - 3 - húmedo , b. Bacteriemia
caliente c. SIRS
d. ICC - 4 – Humedo , frio d. Sepsis foco respiratorio
e. ICC - 3 - Seco , frio . e. Sepsis severa.
20. La insuficiencia respiratoria aguda dura menos 25. Paciente mujer de 40 años ingresa a la
de ……………… horas , y el criterio básico es que emergencia con dolor abdominal, disuria ,
la PaO2 es menor de ………………………..si solo febril
hay esto la IRA es tipo ………………….. PA: 90/50 , FR: 23 x min . se le hace un
a. 24 - 50 mmhg - II urocultivo: E. coli + sensible a amikacina,
b. 48 - 60 mmhg - I diagnostico:
c. 72 - 60 mmhg - II a. Infección
d. 72 - 60 mmhg - I b. Bacteriemia
21. Paciente que ingresa por disnea, alteración del c. SIRS
sensorio , hipertensión endocraneana, edema d. Sepsis
de papila diaforesis el aga muestra: Pa02 : 59 e. Sepsis severa.
mmhg – PaCO2: 60 mmhg PH. 7.10 , HCO3: 30. 26. Paciente diabético hace 20 años usa
a. IRA tipo I - con acidosis metabolica + metformina cada 12 horas esporádicamente,
alcalosis respiratoria compensada ingresa a la emergencia en mal estado general
b. IRA tipo II - con alcalosis metabolica + , somnoliento ( Glasgow 12 ), al examen fisico:
acidosis respiratoria descompensada PA: 100/70 FC: 100 FR: 32 , S02: 90%
c. IRA tipo II – con acidosis metabolica AGA Y ELECTROLITOS: Ph: 7.10 PaO2: 80
+alcalosis respiratoria compensada. PCO2: 25 HCO3. 15 Na: 135 k: 3.5
d. IRA tipo II - con acidosis respiratoria + Glucosa: 800
alcalosis metabolica compensada Puedo afirmar lo siguiente:
e. IRA tipo II - con acidosis respiratoria +
alcalosis metabolica descompensada. a. Es un coma hiperosmolar – osmolaridad de
22. Criterios de hospitalización CURB 65 no 304.4 mOsm /l
considera el siguiente parámetro: b. Es una cetoacidosis diabética –
a. Nitrógeno en sangre osmolaridad 324 ,4 mOsm /l
b. Trastorno del sensorio c. Es un cuadro mixto - osmolaridad sérica
c. Frecuencia respiratoria mas 30 290
d. Presion arterial sistólica o diastólica alta d. Es un síndrome hiperosmolar -
e. Edad mayor de 65. osmolaridad de 314.4 mOsm/l
e. Es una cetoacidosis diabética –
23. Paciente mujer de 70 años, diabético ingresa osmolaridad de 280 mOsm/l
ala emergencia por tos, disnea , PA: 80/60, FC: 27. En el cuadro anterior el tratamiento básico es
130 , Tiraje subcostal mas crepitantes en base …………………………………………………..y el
derecha , fiebre, al hemocultivo sale + para trastorno acido base
Pseudomona sp. UD indicaría el siguiente es………………………………………………………..
tratamiento antibiótico de elección :
a. Solo hidratación con solución isotónica + SO2. 90% - PaO2: 70 mmhg PaCO2.
insulina – acidosis metabólica + alcalosis 45mmhg HCO3: 28 PH: 7.20
respiratoria compensada DIAGNOSTICO:
b. Hidratación con solución hipertónica – a. EPOC LEVE
acidosis respiratoria + alcalosis metabólica b. EPOC MODERARO
compensada c. Asma leve
c. Hidratación + insulina - acidosis d. Asma moderado-grave
metabolica + alcalosis respiratoria e. Asma severo.
descompensada 32. La pregunta anterior que trastorno acido base
d. Insulina – acidosis metabolica + alcalosis tiene su paciente:
respiratoria descompensada a. Alcalosis respiratoria + acidosis metabolica
e. Hidratación + insulina - acidosis compensada
metabolica descompensada. b. Acidosis metabolica + alcalosis respiratoria
c. Acidosis respiratoria mixta
d. Acidosis respiratorio + alcalosis metabolica
28. Paciente varón de 45 años fumador de 10 compensada
cigarrillos por dia, ingresa a la emergencia por e. Acidosis respiratoria + alcalosis metabolica
tos, fiebre, se le realiza un AGA: PH: 7.25 descompensada.
PaCO2: 40 HCO3: 16 . 33. Paciente con síntomas diurnos mas de 1 dia a
Su trastorno acido base que presente este es: la semana, mas de 2 veces al mes de síntomas
a. Acidosis metabolica + alcalosis por la noche, se define como:
respiratoria compensada a. Asma leve
b. Acidosis metabolica compensada b. Asma intermitente leve
c. Acidosis metabólica + acidosis c. Asma persistente leve
respiratoria. d. Asma persistente moderada
d. Alcalosis respiratoria descompensada e. Asma persistente grave
e. Alcalosis respiratoria + acidosis 34. Adulto mayor que estando en centro comercial
metabolica. y próximo a usted, pierde súbitamente la
29. Medicación de control del asma es excepto: conciencia , rápidamente evalúa que no hay
a. Corticoides inhalados riesgo situacional. ¿Cuál es la secuencia correcta
b. Agonistas b adrenérgicos inhalados de su accionar?
c. Corticoides inhalados+ b adrenérgicos de a. verifica que no respira y de ser así dar 2
respiraciones de rescate y luego dar
acción larga
golpe precordial.
d. SABA
b. verificar que no hay pulso, llamar al 116
e. Corticoides orales
y poner la cabeza hacia el lado derecho.
c. evaluar si respira, de no hacerlo, dar
30. Paciente varón con antecedente de ser golpe precordial luego 2 respiraciones y
fumador de 10 cigarrillos por día ingresa ala 15 compresiones por 2 minutos.
emergencia por múltiples reingresos por d. diagnosticar que esta en parada
exacerbaciones en el año, y con recuento de cardiaca y de ser así pedir ayuda e
eosinofilos mas de 300 en sangre, cual es el iniciar 30 compresiones torácicas
tratamiento mas optimo para dicha persona. 35. Ante un adulto al que acaba de diagnosticar
a. SABA + LABA paro cardiorrespiratorio e iniciar reanimación
b. LABA- SAMA cardiopulmonar básica. ¿que otra acción debe
c. SABA + LABA + GCO hacer simultáneamente?
d. LABA+LAMA+ GCI a. Administrar oxigeno y adrenalina
e. LABA + LAMA + GCO b. Infundir adrenalina 5 mg EV
31. Paciente varón de 25 años que ingresa a la c. Proporcionar oxigeno y conectar
emergencia con cuadro de disnea de monitor/desfibrilador
moderada intensidad, usa musculatura d. Cristaloides a goteo
accesoria, ingresa hablando algunas frases. 36. Varón de 54 años que es ingresado a Shock
FR: 25 , FC: 115 , ESPIROMETRIA: VEF1 : 65% trauma, refiriendo : dolor torácico que irradia
hacia el cuello y mandibula, disnea. A los 45
.
minutos se desvanece, apnea, se inicia RCP B. Desfibrilación inmediata
básico y se coloca oxígeno, en el monitor
C. Intubación orotraqueal
cardiaco se visualiza asistolia. En este momento,
D. Compresión cardiaca
según el algortimo: Cuál es la primera acción a
E. Realizar 30 contracciones cardiacas mas 2
seguir
ventilaciones
A) Continuar con compresiones torácicas
B) Administrar adrenalina
Dr ROBINSON LEON ZULOETA
C) Realizar descarga eléctrica
DOCENTE.
D) Mantener vía aérea

37. Número de compresiones ( frecuencia cardiaca )


óptima en el RCP
A) 80 – 90
B) 100 – 120
C) 60 – 80
D) 90 – 100

38. Mujer de 61 años con EPOC acude a emergencia


por presentar mayor disnea, incremento de tos
y de esputo RX de tórax: signos de enfisema sin
opacidades focales. Laboratorio: PaO2 62mmHg.
¿Cuál es la causa de la hipoxemia?
A. Disminución del espacio muerto
B. Aumento del shunt fisiológico
C. Desequilibrio ventilación-perfusión
D. Deterioro de la difusión

39. Varón de 41 años, obeso acude a emergencia


por dolor abdominal difuso de moderada
intensidad, náuseas y vómitos. Al examen físico:
FR: 34 rpm, PA: 110/60 mmHg. Glasgow: 11 pts,
mucosa oral seca. Exámenes de laboratorio:
glucemia: 679 mg/dl, Na+: 145 mEq/l, K+: 3.4
mEq/l, HCO3: 6.5 mEq/l, pH: 6.8, PCO2: 26
mmHg. Cuál es el tratamiento inicial ?

A. Insulina EV inicial, hidratación con solución


hipotónica y gluconato de calcio
B. Hidratación enérgica con solución
isotónica e insulina EV
C. Hidratación enérgica con suero salino
isotónico, insulina EV y bicarbonato EV
D. Solución salina hipotónica, bicarbonato Na
e insulina EV
E. Suero salino isotónico, insulina NPH a dosis
elevadas y bicarbonato EV

40. Varón de 40 años es traído por familiares a


Emergencia e ingresa a Trauma Shock, por
dificultad respiratoria progresiva. Usted observa
que el paciente deja de respirar, no responde al
llamado y no hay pulso carotídeo. ¿Cuál es el
primer paso de actuación?

A. Interconsulta a cardiología
SEMINARIOS AVANZADOS - USS
Examen tipo ENAM
mellitus y obesidad. Ingresa a Emergencia por
dolorprecordial de intensidad 10/10. Al examen
2 EXAMEN TIPO ENAM - SEMINARIOS físico: Palidez, diaforético, frialdad distal y
AVANZADOS AULAS A Y B crepitantes en 1/3 inferior de ambos hemitórax;
ALUMNO : ………………………………………… FC: 60 lat/min; PA: 70/40 mmHg. Hace 2 días
presentó sensación febril y tos no productiva.
AULA ………………… ¿Cuál es el diagnóstico más probable?
a. Shock obstructivo
b. Shock cardiogénico
c. Shock séptico
CARDIOLOGIA
d. Angina inestable
7. En un paciente con shock séptico ¿qué se debe
1. En la fibrilación auricular con respuesta indicar inicialmente?
ventricular rápida, con descompensación a. A. Dopamina B. Fluidos EV
cardíaca, eltratamiento de elección es: C. Coloides
a. Cardioversión eléctrica
b. D. Adrenalina E. Dobutamina
a. Quinidina
b. Lidocaína
c. Digitálico 8. ¿Cuáles la causa más frecuente de shock
cardiogénico?
a. lnsuficiencia mitral aguda.
2. Mujer de 30 años, presenta bruscamente b. lnsuficiencia aórtica aguda.
taquicardia de 200 latidos por minuto, con c. Embolia pulmonar.
severa hipotensión. En el ECG se encuentra P- d. Rofura del tabique
R corto, QRS ancho y de ascenso irregular. ¿El interventricular.
tratamiento inicial indicado es?
e. lnfarto agudo de miocárdio.
a. Verapamilo b. Digoxina
c. Cardioversión
d. Nitroglicerina
9. El mecanismo principal en shock séptico es:
3. El cuadro clínico más característico de la a. Pérdida de líquidos
estenosis aórtica es: b. Disminución de la resistencia
a. Soplo holosistólico, pulso vascular periférica
amplio c. Disminución del gasto cardiaco
d. Hemorragias internas
b. Dolor torácico, disnea y síncope e. Disminución de los factores de
c. Soplo sistólico, pulso saltón y coagulación
disnea

d. Disnea, hemoptisis, edemas 10. Varón de 55 años de edad, presenta dolor


retroesternal súbito e intenso de 6 horas de
4. Varón de 45 años de edad, antecedente de dolor
evolución. El ECG revela la onda Q y segmento ST
retroesternal moderado al esfuerzo e irradiado
elevado en DII, DIII, AVF. ¿Cuál es el diagnóstico
a la región axilar izquierda. Al examen clínico:
y qué fármaco usaría?
PA elevada, taquicardia, ECG: desnivel ST. La
indicacióninmediata en caso de dolor es: a. Tromboembolismo pulmonar/
a. Estreptoquinasa heparina
b. Infarto agudo de
b. Heparina endovenosa miocardio/heparina
c. Internamiento en c. Pleurodinia/ antiinflamatorios no
UCI esteroideos
d. Tromboembolismo pulmonar/
d. Nitroglicerina sublingual trombolítico
5. Varón de 40 años presenta cefalea y mareos. Al e. Infarto agudo de
examen físico: PA 168/80mmHg. IMC:32, miocardio/tombolítico
hallazgo de acantosis nigricans en la región
cervical, aumento de la circunferencia b. NEUMOLOGIA
abdominal. ¿Cuál es el diagnóstico? 11. Paciente varón indigente de 40 años de edad, hallado
a. Hipotiroidismo en la vía pública confuso, es traído a Emergencia.
b. Síndrome metabólico Examen físico presenta lesiones en la lengua,
c. Diabetes respiración profunda y rápida, amaurosis. Lab: pH
mellitus
7.19; pCO2 30mmHg, HCO3: 12. ¿Cuál de los
d. Síndrome de Cushing siguientes es el diagnóstico más probable?
a. Intoxicación por alcohol metílico
6. Varón de 66 años con antecedente de diabetes b. Estado post ictal

12 de noviembre del 2022


Página 1 de 21
SEMINARIOS AVANZADOS - USS
Examen tipo ENAM
c. Neumonía aspirativa medicamento podría ser un ---------- y se considera
d. Insuficiencia respiratoria un ---------, cuyo efecto adverso cardiovascular
probables es --------
a. Beta 2 adrenérgico – agonista total –
12. Varón da 66 años, antecedente de alcoholismo y taquicardia.
drogadicción. Hace 3 semanas fue diagnosticado de b. Beta 2 adrenérgico – agonista parcial -
tuberculosis pulmonar con BK en esputo positivo. taquicardia.
Antes de iniciar el esquema antiTBC, qué exámenes c. Alfa 1 adrenérgico – agonista total –
solicitaría… taquicardia.
a. Hemograma y creatinina d. Alfa 2 adrenérgico – agonista inverso-
b. BK en orina y en jugo gástrico taquicardia.
c. Bilirrubinas y transaminasas e. Beta 2 adrenérgico – agonista inverso –
d. Electrolitos taquicardia

19. Se llama Multidrogoresistente al paciente que es


13. Paciente varón de 36 años con diagnóstico de asma resistente a:
bronquial que presenta síntomas más de una vez por a. Rifampicina y Etambutol
semana, pero menos de una vez al día. El diagnóstico b. Rifampicina y Pirazinamida
correcto sería: c. Rifampicina e Isoniazida
a. Asma persistente moderada d. Rifampicina, Isoniazida y
b. Asma persistente leve Aminoglucósido
c. Asma intermitente e. Rifampicina, Isoniazida y
d. Asma persistente severa Quinolona

14. En un paciente con diagnóstico de EPOC, la 20. ESSALUD 2012 El complejo de Ghon es
espirometría muestra: característico de:
a. Disminución de VEF1 y CVF a. A. Actinomicosis B. LUES congénita
b. Disminución del VEF1 y aumento b. C. Tuberculosis D. Tuberculosis
de CVF primaria
c. Disminución de CFV y VEF1 c. E. Nocardiosis
normal
d. Aumento del VEF1 y disminución
de VR
GASTROENETEROLOGIA
15. Mujer de 82 años con fractura de cadera y postrada,
presenta disnea súbita e hipoxemia. FC=102
lat/min', T 37°C. Si sospecha de TEP. ¿Cuál sería su 21. Mujer de 25 años quien padece de ardor retroesternal
manejo inicial? con episodios de regurgitación e hipersalivación.
a. Trombolíticos ¿Cuál es el diagnóstico probable?
b. Heparina EV a. A. Acalasia B. Divertículo de Zenker
c. Heparina SC b. C. ERGE D. Adenocarcinomade
d. Warfarina esófago

16. Señale el fármaco antimuscarínico broncodilatador 22. Mujer de 40 años, con colelitiasis en lista de espera
que se administra por vía inhalatoria de cirugía, es traída a Emergencia por dolor
a. a. Metaproterenol b. Albuterol abdominal intenso en mesogastrio, coloración
b. c. Ipratropio d. Ritodrina amarillenta de escleras, naúseas y vómitos, los
c. e. Fenotero familiares refieren que hace cuatro días, se le
administró analgésicos con mejoría parcial. Al
examen: PA 120/75 mmHg, FC 86 x´, T 37.8° C,
17. ¿Cuál es el tipo de epitelio que presenta la tráquea?: abdomen doloroso en epigastrio, amilasa en 400,
a. Estratificado cuboidal. b. Estratificado creatinina 0.6mg/dL. ¿Qué examen es mejor para
columnar. descartar complicaciones de la patología en
b. c. Escamoso. d. Cilíndrico sospecha? A. Ecografía de hígado y vía biliar
simple. a. B. Tomografía Abdominal sin y con
c. e. Pseudoestratificado columnar ciliado. contraste
b. C. RMN abdominal
c. D. CTPH

18. Paciente con crisis asmática recibe un medicamento 23. Varón de 57 años acude a Emergencia por heces
que aumenta la actividad basal del receptor que negras, mal olientes, niega vómitos, niega pérdida de
dilata los bronquios a su máxima eficacia, este peso, como antecedente refiere tomar analgésicos

12 de noviembre del 2022


Página 2 de 21
SEMINARIOS AVANZADOS - USS
Examen tipo ENAM
durante 5 días por automedicación . Al examen: profusa. Hematocrito: 18%. ¿Cuál es la indicación
palidez leve, PA 120/75mmHg, FC 78 x´, FR 21x´, inmediata más adecuada?
Sat 98% sin O2 complementario, abdomen no a. Endoscopía alta
doloroso. ¿Cuál es el diagnostico más probable? b. Venoclisis con solución salina a
a. A. Úlcera gástrica C. NM gástrico chorro
b. B. Úlcera duodenal D. NM duodenal c. Transfusión sanguínea
d. Dosaje de tiempo de protrombina
24. De los siguientes enunciados. ¿Cuál se relaciona con e. Radiografía de esófago, estómago
cuadro de colestasis? y duodeno
a. Elevación de bilirrubinas a
predominio indirecto
b. Valores normales de FA y GGT
c. Elevación marcada de AST y
ALT NEFROLOGIA – DERMATOLOGIA
d. Elevación de bilirrubinas
predominio directo, FA y GG
31. Paciente mujer de 71 años de edad con
antecedente de diabetes mellitus ingresa por dolor
25. Varón de 56 años antecedente de gastrectomía por abdominal mas vomitos con un peso de 68 kg, a los
NM gástrico. ¿Cuál es la anemia que puede exámenes de sangre: creatinina: 1.45 , cual es la
presentar a largo plazo? tasa de filtración glomerular: ml / min.
a. A. Anemia megaloblástica C. a. 13.70
Anemia hemolítica b. 23.79
b. B. Anemia por deficiencia de folatos D. c. 33.70
Anemia por enfermedad crónica d. 43.70
e. 44.94

26. La principal complicación de la pancreatitis aguda


es: 32. En el paciente anterior en que grado de falla renal
a. Diabetes mellitus crónica se encuentra el paciente:
b. Pseudoquiste pancreático a. I
c. Desnutrición b. II
d. Ictericia c. III
e. Esteatorrea d. IV
e. V
27. ¿En la pancreatitis aguda ¿Cuál de los siguientes
exámenes es de valor diagnóstico precoz?
a. A. Amilasa B. Transaminasa 33. El diagnóstico de infección urinaria a partir de una
b. C. Lipasa D. Fórmula y recuento muestra por chorro medio de orina, lo da el
leucocitario crecimiento de:
c. E. Proteína e reactiva a. 10 5 colonias por litro
b. 10 3 colonias por mL
c. 10 5 colonias por mL
28. Los criterios de Ranson en pancreatitis aguda d. 10 3 colonias por litro
establecen: e. 10 2 colonias por Ml
a. Evolución de la enfermedad
b. Complicaciones 34. Mujer de 28 años que acude a consulta por cuadro
c. Pronóstico agudo de artralgias, fiebre, náuseas y vómitos. Al
d. D. Diagnóstico examen presenta edemas (+/+++), PA 180/110
e. E. Tipo patológico mmHg, el estudio de orina muestra proteínas 3(+),
abundantes glóbulos rojos crenados, cilindros
29. Aproximadamente el 90% de los pacientes con hemáticos, leucocitarios y granulosos. Urea: 80
hematozequia sangra por el: mg/dl, Creatinina 3 mg/dl y complemento C3 y C4
a. A. Recto B. Esófago bajos. ¿Cuál es el diagnóstico probable?
b. C. Estómago D. Duodeno a. A. Pielonefritis aguda B. Síndrome
c. E. Colon nefrótico
b. C. Nefritis lúpica D. Nefritis
tubulointersticial
30. Varón de 46 años de edad, que acude a Emergencia c. E. Nefropatía por IgA
por lipotimia posterior a hematemesis y melena de
aproximadamente 800 mL. Al examen físico:
taquicardia, palidez, hipotensión y sudoración

12 de noviembre del 2022


Página 3 de 21
SEMINARIOS AVANZADOS - USS
Examen tipo ENAM
35. La lesión histopatológica característica de la con aumento de volumen en forma difusa. Corazón:
glomerulonefritis rápidamente progresiva es: ruídos cardíacos taquicárdicos. ¿Cuál es el
a. Proliferación mesangial diagnóstico probable?
b. Engrosamiento de la membrana a. A. Tiroiditis de Hashimoto C.
basal Enfermedad de Graves Basedow
c. Compromiso glomerular focal y b. B. Tiroiditis subaguda D.
segmentario Enfermedad de Plummer
d. Proliferación epitelial
extracapilar
e. Proliferación endotelial 42. Paciente varón de 35 años es traído a Emergencia
con trastorno del sensorio, obnubilado, polipneico,
36. La Psoriasis inversa, afecta principalmente: se evidencia mucosas orales secas. Estudio de
a. Dorso de la mano b. Codos Glicemia 880 mg/dL, pH =7.20, HCO3=13, pCO2
b. c. Rodillas d. Axilas =24, cetonuria (++), Na+ sérico:145. ¿Cuál es el
c. e. Cuello manejo ideal?
a. Hidratación con agua destilada,
insulina EV.
37. La Pseudomonas aeruginosa es causante de una de b. Hidratación enérgica con suero
las siguientes enfermedades dermatológicas fisiológico, insulina en bolo y en
a. a. Ectima ampollar b. Ectima vulgar infusión.
b. c. Ectima contagioso d. Ectima c. Hidratación con suero
gangrenoso hipotónico, bicarbonato de sodio
c. e. Erisipeloide EV.
d. Insulina EV en bolo y
bicarbonato de sodio.

38. El impétigo ampolloso es causado por: 43. . ¿Cuál es el valor normal de glucosa en mg/dL a 2
a. Staphylococcus aureus coagulasa negativo horas después de una sobrecarga oral de 75g de
b. Streptococcus pneumoniae glucosa anhidra?
c. Hemophilus influenzae a. Menos de 140
d. Staphylococcus aureus coagulasa positivo b. Menos de 180
e. Pseudomonas aeruginosa c. Menos de 120
d. Menos de 100

39. Lesión descamativa plateada, sobre una mancha


roja. ¿Cuál es el diagnóstico más probable 44. Varón de 30 años con antecedente de TBC
a. Dermatitis seborreica pulmonar, llega a la Emergencia por deshidratación
b. Liquen simple crónico asociado a vómitos e hipotensión, además de
c. Psoriasis fiebre. Al examen físico: presenta shock,
d. Tiña corporis hiperpigmentación en labios y pliegues cutáneos.
e. Liquen plano Electrolitos séricos: Hiponatremia. ¿Cuál es su
presunción diagnóstica?
40. El diagnóstico de ectima estreptocócico se debe a. Hiperaldosteronismo primario
sospechar cuando se presentan una o más úlceras b. Shock distributivo
induradas principalmente en: c. Hiperaldosteronismo secundario
a. a. Región facial b. Región d. Crisis adrenal
cervical
b. c. Miembros inferiores d. Región 45. Paciente mujer de 22 años, peso 86 kilos, talla 150
torácica centímetros además con fascie pletórica,
c. e. Miembros superiores hirsutismo, debilidad muscular proximal. En las
funciones vitales: PA. 166/100mmHg, glucosa en
ayunas 120 mg/dL. El diagnóstico más probable es:
a. Obesidad exógena
ENDOCRINOLOGIA - INFECTOLOGIA b. Diabetes mellitus tipo II
c. Polimiositis
d. Enfermedad de Cushing
41. Mujer de 38 años, desde hace 10 meses refiere
nerviosismo, intolerancia al calor, cansancio, 46. Paciente mujer de 18 años es traída a la EMG por
labilidad emocional y pérdida de peso. Al examen presentar trastorno de conciencia, se toma prueba
físico: disminución de peso, habla rápido, rápida de glucosa resultando en 59 mg/dL. En el
exoftalmos evidente. Cuello: a la palpación tiroides

12 de noviembre del 2022


Página 4 de 21
SEMINARIOS AVANZADOS - USS
Examen tipo ENAM
manejo inicial, la dextrosa al cuánto por ciento se a. Intoxicación por alcohol metílico
da: C. Estado post ictal
a. A. 10% C. 5% b. B. Neumonía aspirativa D.
b. B. 33% D. 20% Insuficiencia respiratoria

47. Neonato que al examen físico presenta peso y talla 52. Varón de 50 años es traído a la Emergencia por
normal. Se observa piel fría, cabello aspero y pérdida de conciencia hace 30 minutos. En Shock
fontanelas abiertas, ojos separados, macroglosia y trauma presenta convulsiones, con antecedente de
lesión de hernía umbilical. ¿Cuál es el diagnóstico hipertensión arterial mal controlada. Examen
más probable? Físico: PA 180/100 mmHg. FC:50 lat/min, FR 30,
a. A. Hipopituitarismo B. Sífilis SatO2 80%, FIO2: 21%; anisocoria y signo de
b. C. Hiperplasia suprarrenal D. Sindrome de decorticación. ¿Cuál es el diagnóstico más
Cri Du Chat probable?
c. E. Hipotiroidismo a. ACV isquémico B. ACV
hemorrágico
48. Primigesta de 28 años de edad que hace un mes b. C. HSA D. Malformación
presenta úlcera genital en el lado derecho, indolora. vascular
Niega otras molestias. Examen ginecológico: labio
mayor derecho: úlcera de 0,5 x 0,5 cm, incdolora, 53. Mujer de 52 años, mientras realizaba sus
de color grisácea, no adenopatía inguinal. RPR: actividades en casa presenta de manera súbita
1/180. El tratamiento indicado es: cefalea muy intensa, asociada a vómitos y luego
a. Penicilina benzatínica 1.200.000 pérdida de conciencia. Al examen físico presenta
UI IM, 1 amp/sem x 3 sem rigidez de nuca. ¿Cuál es el diagnóstico más
b. Penicilina benzatínica 2.400.000 probable?
UI IM, 1 amp/sem x 3 sem a. A. Tumor cerebral C. Hemorragia
c. Penicilina benzatínica 1.000.000 subaracnoidea
UI IM, 1 amp/sem x 3 sem b. B. TIA D. ACV
d. Penicilina sódica, 80.000 UI, IM isquémico
1 amp/sem x 3 sem
e. Penicilina sódica, 100.000 UI, IM 54. . Paciente mujer de 25 años, presenta cese brusco
1 amp/sem x 3 sem de sus actividades, mirada fija y movimiento
repetido del brazo derecho como para coger algo, la
49. Mujer con lesiones vesiculares en los labios duración fue de 5 minutos y no recuerda nada.
mayores asociadas a adenopatías inguinales. ¿Cuál ¿Cuál es el diagnóstico?
es el diagnóstico más probable? a. A. Crisis parcial simple C. Crisis
a. A. Herpes Zoster B. Chancroide tónico-clónicas
b. C. Herpes genital D. Gonorrea b. B. Crisis parcial compleja D. Crisis de
c. E. Sifilis ausencia atípica

50. Varón con descarga uretral mucosa y luego 55. Varón de 30 años, fumador. Presenta intenso dolor
purulenta ¡Cuál es el agente etiológico más periorbitario asociado a lagrimeo y congestión
probable? conjuntival. El cuadro se exacerba con ingesta de
a. A. Mycoplasma gentalium B. licor. Las crisis duran aproximadamente 20
Haemophilus vaginalis minutos y pueden repetirse durante el día. ¿Cuál es
b. C. Chlamydia trachomatis D. Gonococo el diagnóstico más probable?
c. E. Thricomonas vaginalis a. Cefalea tipo migraña C. HSA
b. Cefalea tensional D. Cefalea en
racimos

56. Paciente mujer de 28 años, hace 10 días presentó


NEUROLOGIA cuadro de Influenza. Desde ayer parestesias en
miembros inferiores y debilidad progresiva que la
imposibilita deambular. Presenta pérdida de fuerza
51. Paciente varón indigente de 40 años de edad, en miembros superiores además de dificultad
hallado en la vía pública confuso, es traído a respiratoria. Al examen: cuadriparesia hipotónica e
Emergencia. Examen físico presenta lesiones en la hiporreflexia.
lengua, respiración profunda y rápida, amaurosis. a. El diagnóstico probable es:
Lab: pH 7.19; pCO2 30mmHg, HCO3: 12. a. Síndrome de Guillain-Barré
a. ¿Cuál de los siguientes es el diagnóstico b. Polimiositis
más probable? c. Mielitis transversa
d. Polineuropatía carencial

12 de noviembre del 2022


Página 5 de 21
SEMINARIOS AVANZADOS - USS
Examen tipo ENAM
57. Escolar de 14 años de edad es traído a Emergencia
por presentar parálisis que se inicia en el miembro
inferior izquierdo, que luego se generaliza en todo 62. ¿Cuál es el anticuerpo con alta probabilidad para el
el hemicuerpo del mismo lado. El paciente diagnóstico de Artritis reumatoide superior al factor
proviene de la zona de Cerro de Pasco. El reumatoide?
diagnóstico más probable es: a. A. Anti-CCP C. ANA
a. A. MEC C. Hidrocefalia b. B. AntiDNA D. PCR 50.
b. B. Neurocisticercosis D. Epilepsia
63. Mujer de 21 años hace 4 meses presenta malestar
general, poliartritis de manos. Al examen físico:
PA: 130/80 mmHg, FC: 103 x minuto. FR: 28 x
58. Niño de 2 años de edad, con diagnóstico clínico de minuto. Alopecia, palidez y adenopatías cervicales
meningitis. El LCR muestra cocobacilos múltiples. Hemoglobina 7 g/dL, Úrea 70 mg/dL,
gramnegativos intra y extracelulares. ¿En cuál Creatinina 2.5 mg/dL. Examen de orina:
microorganismo sospecharía?: Hematuria, cilindros hemáticos y proteínas (+),
a. a. Escherichia coli b. Hemophilus ANA y anti- Smith (+). Rx de pulmones: Lesión
influenzae homogénea en el tercio inferior del hemitórax
b. Klebsiella pneumoniae d. Listeria izquierdo. ¿Cuál es el diagnóstico más probable?
monocytogenes a. Artritis reumatoide
c. e. Streptococcus pneumoniae b. Síndrome de Goodpasture
c. Lupus eritematoso sistémico
59. Mujer de 45 años refiere episodios de dolor d. Tuberculosis sistémica
paroxístico en la arcada dentaria superior derecha
desde hace dos meses que ha ido aumentando en
frecuencia e intensidad. El dolor es como si HEMATOLOGIA
recibiera descargas eléctricas que desde la región
malar descienden y terminan en las encías. NO 64. El marcador más importante para el diagnóstico de
cede con analgésicos comunes. Fue evaluada en anemia carencial crónica es …
odontología y no encontraron alteraciones. Al A. Ferritina
examen no presenta defectos sensitivos, pero el b. Hemosiderina
dolor se desencadena al tocar las encías o la región c. Reticulocitos
geniana derecha. Según su criterio clínica, ¿cuál es D. Transferrina
el tratamiento inicial que debe recibir la paciente?:
a. a. Carbamazepina b.Amitriptilina 65. Varón de 62 años, con cardiopatía isquémica,
b. c. Tramadol d.Ketorolaco acude por dolor en región precordial irradiado a
c. e. Codeína región cervical. Al examen físico: taquicárdico,
palidez ++/+++. Laboratorio: HB: 8 g/dL, glucosa:
80mg/dL, creatinina normal. ¿Cuál es el manejo
60. ¿En cuál de las siguientes entidades NO se ideal?
encuentra rigidez de nuca? a. Transfusión de paquete globular
a. Artrosis cervical b. b. Hierro oral
Meningitis c. Indicar fierro parenteral
b. Hipertensión endocraneana severa d. d. Observación y repetir
Coma hipoglicémico hemoglobina
c. e. Hemorragia subaracnoidea

66. Varón de 56 años antecedente de gastrectomía por


NM gástrico. ¿Cuál es la anemia que puede
presentar a largo plazo?
REUMATOLOGIA a. A. Anemia megaloblástica
b. Anemia hemolítica
c. Anemia por deficiencia de folatos
61. Mujer de 30 años que desde hace 5 meses refiere d. Anemia por enfermedad crónica
dolor y edema en ambas manos. Examen físico:
Dolor con aumento de volumen en articulaciones
metacarpofalángicas proximales y desviación 67. Varón de 78 años con antecedente dolor lumbar y
cubital. ¿Cuál es el diagnóstico más probable? de extremidades al deambular además de astenia de
a. A. Lupus eritematoso sistémico C. cinco meses de evolución. Al examen: PA: 140/80
Artritis gotosa mmHg, FC:78 lat/minuto, FR: 20 por minuto.
b. B. Artritis infecciosa D. Palidez ++/+++, hepatoesplenomegalia.
Artritis reumatoide

12 de noviembre del 2022


Página 6 de 21
SEMINARIOS AVANZADOS - USS
Examen tipo ENAM
Laboratorio: anemia, hipercalcemia, azoemia.
¿Cuál es el diagnóstico más probable?
a. A. Leucemia mieloide crónica C.
Mieloma múltiple
b. B. Leucemia de células plasmáticas D.
Leucemia linfática crónica

68. Mujer de 65 años de edad que consulta por palidez,


fatiga y palpitaciones al esfuerzo desde hace dos
meses. Glicemia: 98 mg%, creatinina, 1,6mg%,
hematocrito 21%, VCM de 68 um3, HCM de 21
pg/célula, CHCM de 27 g/dL, amplitud de
distribución eritrocitaria(RDW) 21%. ¿Cuál de los
siguientes tipos de anemia es la más probable?
a. A. Refractaria B. Aplásica
b. C. Perniciosa D. Por insuficiencia
renal crónica
c. E. Ferropénica

69. Varón de 16 años de edad, de raza negra. Acude


por consultorio externo debido a palidez y dolores
óseos. Al examen físico presenta esplenomegalia.
Hemoglobina: 8 g/dL. ¿Cuál es el tipo de anemia
más probable?:
a. A. Perniciosa B. Aplásica
b. C. Drepanocítica D. Déficit de folatos
c. E. Ferropénica

70. Paciente de 70 años de edad, con anemia crónica


severa. VCM: 101 fL y HCM: 25 pg. Considerando
estos valores. ¿Cómo clasifica a la anemia?
a. Microcítica normocrómica
crónica B. Microcítica
hipocrómica
b. C. Normocítica normocrómica D.
Macrocítica hipocrómica
c. E. Macrocítica normocrómica

71. Niño con pancitopenia y visceromegalia ¿Cuál es el


diagnóstico más probable?
a. Leucemia mielomielocítica
crónica
b. Leucemia mieloide aguda.
c. Leucemia linfática aguda
d. Leucemia mielógena crónica
e. Leucemia linfocítica crónica

ROBINSON LEON ZULOETA

DOCENTE.

12 de noviembre del 2022


Página 7 de 21
Examen tipo ENAM MDSURG Academy
Academy

1. Mujer de 45 años que hace 3 meses presenta dolores articulares intermitentes que ceden a los
analgésicos. Ha notado baja de peso, sensación de alza térmica vespertina con sudoración parcelar.
Hace 5 días dolor en articulación sacro-iliaca que limita la marcha. ¿Cuál es el probable diagnóstico?
A. Borreliosis C. TBC articular
B. Salmonelosis D. Brucelosis

2. Paciente de 15 años procedente de Jaén, presenta fatiga, palpitaciones, tos con expectoración
hemoptoica y diarreas. Al examen: piel y mucosas pálidas, abdomen distendido, edema y
escoriaciones de miembros inferiores. ¿Cuál es la presunción diagnóstica?
A. Estrongiloidiasis C. Fascioliasis
B. Ascaridiasis D. Anquilostomiasis

3. ¿Cuál es la opción incorrecta?:


A. Chancro duro, Treponema pallidum; úlcera no dolorosa
B. Chancro blando, Haemophilus ducreyi; úlcera dolorosa
C. Linfogranuloma venéreo, Chlamydia trachomatis; úlcera no dolorosa
D. Condiloma plano, Neisseria gonorrhoeae; secreción uretral purulenta

4. La Prueba de Tzanck es una tinción que revela la presencia de células gigantes multinucleadas y es
útil en el diagnóstico de infecciones virales por:
A. Adenovirus C. Influenza virus
B. Herpes virus D. Coronavirus

5. Varón de 50 años, alcohólico, ingresa por Emergencia con fiebre de 39ºC, tos y expectoración fétida.
Al examen respiratorio hay disminución del MV en hemitórax derecho. La radiografía de tórax
muestra un absceso con nivel hidroaéreo en el segmento 6 derecho. ¿Cuál es el germen más
probable?
A. Staphylococcus aureus C. Klebsiella pneumoniae
B. Bacterias anaerobias D. Mycobacterium tuberculosis

6. Paciente con sospecha de Neurosífilis. ¿Cuál es el examen diagnóstico que hay que realizar en este
paciente?
A. FTA-ABS en LCR C. Test de Nelson
B. Examen de campo oscuro de LCR D. VDRL en LCR

7. Paciente gestante con tuberculosis pulmonar activa. ¿Cuál es el régimen de tratamiento que
recomendaría?
A. Rifampicina, Isoniacida, Pirazinamida C. Rifampicina, Isoniacida, Estreptomicina
B. Rifampicina, Isoniacida, Etambutol D. Isoniacida, Pirazinamida, Etambutol

8. La causa más frecuente de infección secundaria del SNC en pacientes con SIDA es:
A. Criptococosis meníngea C. Toxoplasmosis cerebral
B. Encefalitis por CMV D. Leucoencefalopatía multifocal

9. Paciente con hiperpotasemia sintomática, debilidad muscular, electrocardiograma con QRS ancho y
sin ondas P. ¿Cuál es el fármaco inicial que utilizaría?
A. Kayexalate C. B2 agonistas
B. Insulina con glucosa D. Gluconato de calcio

10. En la fibrilación auricular con respuesta ventricular rápida, sin descompensación cardíaca, el
tratamiento de elección es:
A. Cardioversión eléctrica C. Quinidina
B. Lidocaína D. Digitálico

12 de noviembre del
del 2022
2022
Página
Página 23 de
de 21
21
Examen tipo ENAM MDSURG Academy

11. Mujer de 30 años, presenta bruscamente taquicardia de 200 latidos por minuto, con severa
hipotensión. En el ECG se encuentra P-R corto, QRS ancho y de ascenso irregular. ¿El tratamiento
inicial indicado es?
A. Verapamilo C. Digoxina
B. Cardioversión D. Nitroglicerina

12. El cuadro clínico más característico de la estenosis aórtica es:


A. Soplo holosistólico, pulso amplio C. Dolor torácico, disnea y síncope
B. Soplo sistólico, pulso saltón y disnea D. Disnea, hemoptisis, edemas

13. Paciente evaluado en Emergencia, en el ECG se evidencia bradicardia y alargamiento progresivo de


PR, estamos frente a un caso de:
A. Mobitz I C. Bloqueo de alto grado
B. Mobitz II D. Primer grado

14. En un paciente con ICC, el galope S3 está relacionado con:


A. Contraccion auricular C. Llenado ventrícular rápido
B. Llenado aurícular rápido D. La presístole

15. Varón de 45 años de edad, antecedente de dolor retroesternal moderado al esfuerzo e irradiado a
la región axilar izquierda. Al examen clínico: PA elevada, taquicardia, ECG: desnivel ST. La indicación
inmediata en caso de dolor es:
A. Estreptoquinasa C. Heparina endovenosa
B. Internamiento en UCI D. Nitroglicerina sublingual

16. ¿Cuál es el trastorno ácido-base en una paciente con la siguiente gasometría arterial: pH:7?19,
HCO3:14 mEq/L, pCO2: 34 mmHg?
A. Acidosis respiratoria C. Acidosis mixta
B. Acidosis metabólica compensada D. Acidosis metabólica

17. Varón de 50 años con 3 días de evolución de diarrea intensa y deshidratación severa, diuresis menor
de 0.5ml/kg por más de 6 horas. Al examen: PA. 90/40mmHg FC: 102 lat/min FR:28 resp/min. ¿Cuál
es el diagnóstico más probable?
A. IRC estadio 3 C. Insuficiencia post renal
B. IRC estadio 4 D. Insuficiencia prerrenal

18. Escolar de 8 años es traída por Emergencia por presentar anasarca, oliguria, palidez en piel y
mucosas y dolor abdominal. Al examen de orina: Proteínas (+++), hematíes (-). Proteínas séricas
totales: 3.5 gr/dL, albúmina: 3 gr/dL, colesterol y triglicéridos elevados. ¿Cuál es el diagnóstico?
A. Síndrome nefrítico C. Síndrome nefrótico
B. Insuficiencia renal aguda D. Desnutrición severa

19. Varón de 89 años, es traído a Shock trauma de emergencia por presentar de manera súbita
episodio convulsivo con relajación de esfínteres. Al examen físico: Hemodinámicamente estable, no
edema. Al examen laboratorial: Sodio sérico en 109 mEq/L. En el manejo la reposición de sodio con
solución salina es al:
A. 9% C. 3%
B. 5% D. 10%

20. Paciente de 6 años hace 10 días presenta faringoamigdalitis lo cual recibió tratamiento con
paracetamol. Es traída por su madre ya que actualmente presenta hematuria y pérdida de proteínas
500mg diario. ¿Cuál es el diagnóstico?
A. Glomerulonefritis membranosa C. IRA
B. Sd. Nefrótico D. Glomerulonefritis postestreptocócica

21. En el esquema de tratamiento de la tuberculosis. ¿Cuál fármaco se asocia a neuritis retrobulbar?


A. Isoniacida C. Rifampicina
B. Pirazinamida D. Etambutol

12 de noviembre del 2022


Página 4 de 21
Examen tipo ENAM MDSURG Academy

22. Varón da 66 años, antecedente de alcoholismo y drogadicción. Hace 3 semanas fue diagnosticado
de tuberculosis pulmonar con BK en esputo positivo. Antes de iniciar el esquema antiTBC, qué
exámenes solicitaría…
A. Hemograma y creatinina C. BK en orina y en jugo gástrico
B. Bilirrubinas y transaminasas D. Electrolitos

23. Paciente varón de 36 años con diagnóstico de asma bronquial que presenta síntomas más de una
vez por semana, pero menos de una vez al día. El diagnóstico correcto sería:
A. Asma persistente moderada C. Asma persistente leve
B. Asma intermitente D. Asma persistente severa

24. En un paciente con diagnóstico de EPOC, la espirometría muestra:


A. Disminución de VEF1 y CVF C. Disminución del VEF1 y aumento de CVF
B. Disminución de CFV y VEF1 normal D. Aumento del VEF1 y disminución de VR

25. Varón de 25 años, desde hace 14 días presenta cefalea, tos no productiva, SAT no cuantificada y
mal estado general. El examen físico preferencial pulmonar es normal. Examen laboratorio:
Hemograma normal. Radiografía de tórax: infiltrado intersticial bilateral en bases. El agente
etiológico más probable es:
A. Mycoplasma pneumoniae C. Klebsiella pneumoniae
B. Streptococcus pneumoniae D. Haemophilus influenzae

26. Paciente varón de 70 años, es hospitalizada en UCI por intoxicación por benzodiazepinas se
encuentra en destete de ventilación mecánica. Al cuarto día presenta cambio de las características
e incremento de secreciones bronquiales, asociada a fiebre y shock. En la radiografía de tórax:
infiltrado alveolar difuso. El tratamiento antibiótico empírico ideal es:
A. Amikacina-ceftazidima-ciprofloxacina C. Ceftriaxona-azitromicina-amikacina
B. Meropenem-vancomicina-amikacina D. Ceftazidima-oxacilina-amikacina

27. Mujer de 82 años con fractura de cadera y postrada, presenta disnea súbita e hipoxemia. FC=102
lat/min', T 37°C. Si sospecha de TEP. ¿Cuál sería su manejo inicial?
A. Trombolíticos C. Heparina EV
B. Heparina SC D. Warfarina

28. Mujer de 38 años, desde hace 10 meses refiere nerviosismo, intolerancia al calor, cansancio,
labilidad emocional y pérdida de peso. Al examen físico: disminución de peso, habla rápido,
exoftalmos evidente. Cuello: a la palpación tiroides con aumento de volumen en forma difusa.
Corazón: ruídos cardíacos taquicárdicos. ¿Cuál es el diagnóstico probable?
A. Tiroiditis de Hashimoto C. Enfermedad de Graves Basedow
B. Tiroiditis subaguda D. Enfermedad de Plummer

29. Paciente varón de 35 años es traído a Emergencia con trastorno del sensorio, obnubilado,
polipneico, se evidencia mucosas orales secas. Estudio de Glicemia 880 mg/dL, pH =7.20, HCO3=13,
pCO2 =24, cetonuria (++), Na+ sérico:145. ¿Cuál es el manejo ideal?
A. Hidratación con agua destilada, insulina EV.
B. Hidratación enérgica con suero fisiológico, insulina en bolo y en infusión.
C. Hidratación con suero hipotónico, bicarbonato de sodio EV.
D. Insulina EV en bolo y bicarbonato de sodio.

30. ¿Cuál es el valor normal de glucosa en mg/dL a 2 horas después de una sobrecarga oral de 75g de
glucosa anhidra?
A. <140 C. <100
B. <180 D. <120

12 de noviembre del 2022


Página 5 de 21
Examen tipo ENAM MDSURG Academy

31. Varón de 30 años con antecedente de TBC pulmonar, llega a la Emergencia por deshidratación
asociado a vómitos e hipotensión, además de fiebre. Al examen físico: presenta shock,
hiperpigmentación en labios y pliegues cutáneos. Electrolitos séricos: Hiponatremia. ¿Cuál es su
presunción diagnóstica?
A. Hiperaldosteronismo primario C. Shock distributivo
B. Hiperaldosteronismo secundario D. Crisis adrenal

32. Paciente mujer de 22 años, peso 86 kilos, talla 150 centímetros además con fascie pletórica,
hirsutismo, debilidad muscular proximal. En las funciones vitales: PA. 166/100mmHg, glucosa en
ayunas 120 mg/dL. El diagnóstico más probable es:
A. Obesidad exógena C. Diabetes mellitus tipo II
B. Polimiositis D. Enfermedad de Cushing

33. Paciente mujer de 18 años es traída a la EMG por presentar trastorno de conciencia, se toma prueba
rápida de glucosa resultando en 59 mg/dL. En el manejo inicial, la dextrosa al cuánto por ciento se
da:
A. 10% C. 5%
B. 33% D. 20%

34. Varón de 40 años presenta cefalea y mareos. Al examen físico: PA 168/80mmHg. IMC:32, hallazgo
de acantosis nigricans en la región cervical, aumento de la circunferencia abdominal. ¿Cuál es el
diagnóstico?
A. Hipotiroidismo C. Síndrome metabólico
B. Diabetes mellitus D. Síndrome de Cushing

35. Varón de 66 años con antecedente de diabetes mellitus y obesidad. Ingresa a Emergencia por dolor
precordial de intensidad 10/10. Al examen físico: Palidez, diaforético, frialdad distal y crepitantes en
1/3 inferior de ambos hemitórax; FC: 60 lat/min; PA: 70/40 mmHg. Hace 2 días presentó sensación
febril y tos no productiva. ¿Cuál es el diagnóstico más probable?
A. Shock obstructivo C. Shock cardiogénico
B. Shock séptico D. Angina inestable

36. Paciente varón de 45 años con antecedente de alcoholismo, acude a Emergencia por presentar
hematemesis y melena profusa. Al examen PA.: 70/40 mmHg, FC: 140 lat/min, palidez marcada,
polipneico, sudoroso y caída del débito urinario. El manejo inicial inmediato es:
A. Dos catéteres venosos periféricos y CINa 0,9% C. Paquete globular por vía periférica
B. Catéter venoso central y paquete globular D. Catéter venoso central y cristaloides

37. Paciente varón de 70 años con antecedente de fibrilación auricular, es llevado a EMG por pérdida
del estado de conciencia. En el examen físico: no se palpa pulso arterial carotídeo. Se inicia RCP, el
monitor evidencia fibrilación ventricular. ¿Cuál es la conducta inmediata?
A. Administrar amiodarona IV C. Desfibrilación
B. Administrar epinefrina IV D. Sólo masaje cardíaco

38. Varón de 15 años que ingresa a Emergencia por intento de suicidio. Al examen se evidencia
fasciculaciones, sequedad de lengua y mucosas, midriasis y taquicardia. ¿A qué tipo de efecto
farmacológico corresponde?
A. Bloqueador colinérgico C. Bloqueador adrenérgico
B. Agonista colinérgico D. Parasimpaticomimético

39. Es traída a Emergencia una niña de 3 años que había estado jugando en la chacra. Posteriormente
presenta sialorrea, debilidad, vómitos, diarrea y cólicos. Examen físico: fasciculaciones musculares
y pupilas mióticas. ¿Cuál es el tratamiento ideal?
A. Flumazenilo C. N-acetilcisteína
B. Sulfato de atropina D. Fisostigmina

12 de noviembre del 2022


Página 6 de 21
Examen tipo ENAM MDSURG Academy

40. Paciente varón indigente de 40 años de edad, hallado en la vía pública confuso, es traído a
Emergencia. Examen físico presenta lesiones en la lengua, respiración profunda y rápida, amaurosis.
Lab: pH 7.19; pCO2 30mmHg, HCO3: 12. ¿Cuál de los siguientes es el diagnóstico más probable?
A. Intoxicación por alcohol metílico C. Estado post ictal
B. Neumonía aspirativa D. Insuficiencia respiratoria

41. Varón de 50 años es traído a la Emergencia por pérdida de conciencia hace 30 minutos. En Shock
trauma presenta convulsiones, con antecedente de hipertensión arterial mal controlada. Examen
Físico: PA 180/100 mmHg. FC:50 lat/min, FR 30, SatO2 80%, FIO2: 21%; anisocoria y signo de
decorticación. ¿Cuál es el diagnóstico más probable?
A. ACV C. HSA
isquémico D. Malformación vascular
B. ACV
hemorrágico

42. Mujer de 52 años, mientras realizaba sus actividades en casa presenta de manera súbita cefalea
muy intensa, asociada a vómitos y luego pérdida de conciencia. Al examen físico presenta rigidez
de nuca. ¿Cuál es el diagnóstico más probable?
A. Tumor cerebral C. Hemorragia subaracnoidea
B. TIA D. ACV isquémico

43. Paciente mujer de 25 años, presenta cese brusco de sus actividades, mirada fija y movimiento
repetido del brazo derecho como para coger algo, la duración fue de 5 minutos y no recuerda nada.
¿Cuál es el diagnóstico?
A. Crisis parcial simple C. Crisis tónico-clónicas
B. Crisis parcial compleja D. Crisis de ausencia atípica

44. Varón de 30 años, fumador. Presenta intenso dolor periorbitario asociado a lagrimeo y congestión
conjuntival. El cuadro se exacerba con ingesta de licor. Las crisis duran aproximadamente 20
minutos y pueden repetirse durante el día. ¿Cuál es el diagnóstico más probable?
A. Cefalea tipo migraña C. HSA
B. Cefalea tensional D. Cefalea en racimos

45. Paciente mujer de 28 años, hace 10 días presentó cuadro de Influenza. Desde ayer parestesias en
miembros inferiores y debilidad progresiva que la imposibilita deambular. Presenta pérdida de
fuerza en miembros superiores además de dificultad respiratoria. Al examen: cuadriparesia
hipotónica e hiporreflexia. El diagnóstico probable es:
A. Síndrome de Guillain-Barré C. Polimiositis
B. Mielitis transversa D. Polineuropatía carencial

46. Escolar de 14 años de edad es traído a Emergencia por presentar parálisis que se inicia en el
miembro inferior izquierdo, que luego se generaliza en todo el hemicuerpo del mismo lado. El
paciente proviene de la zona de Cerro de Pasco. El diagnóstico más probable es:
A. MEC C. Hidrocefalia
B. Neurocisticercosis D. Epilepsia

47. Varón de 20 años de edad mientras manejaba bicicleta sufre traumatismo en tercio inferior de
pierna derecha, ocasionándole excoriaciones. Tres días después presenta dolor y tumefacción en
rodilla derecha. Se realiza artrocentesis: hallazgo líquido sinovial purulento. ¿Cuál es el agente
etiológico más probable?
A. Staphylococcus epidermidis C. Staphylococcus aureus
B. Streptococcus pneumoniae D. E. coli

48. Mujer de 30 años que desde hace 5 meses refiere dolor y edema en ambas manos. Examen físico:
Dolor con aumento de volumen en articulaciones metacarpofalángicas proximales y desviación
cubital. ¿Cuál es el diagnóstico más probable?
A. Lupus eritematoso sistémico C. Artritis gotosa
B. Artritis infecciosa D. Artritis reumatoide

12 de noviembre del 2022


Página 7 de 21
Examen tipo ENAM MDSURG Academy

49. ¿Cuál es el anticuerpo con alta probabilidad para el diagnóstico de Artritis reumatoide superior
al factor reumatoide?
A. Anti-CCP C. ANA
B. AntiDNA D. PCR

50. Mujer de 21 años hace 4 meses presenta malestar general, poliartritis de manos. Al examen
físico: PA: 130/80 mmHg, FC: 103 x minuto. FR: 28 x minuto. Alopecia, palidez y adenopatías
cervicales múltiples. Hemoglobina 7 g/dL, Úrea 70 mg/dL, Creatinina 2.5 mg/dL. Examen de
orina: Hematuria, cilindros hemáticos y proteínas (+), ANA y anti- Smith (+). Rx de pulmones:
Lesión homogénea en el tercio inferior del hemitórax izquierdo. ¿Cuál es el diagnóstico más
probable?
A. Artritis reumatoide C. Síndrome de Goodpasture
B. Lupus eritematoso sistémico D. Tuberculosis sistémica

51. Paciente mujer de 52 años, procedente de Jayanca, acude a consulta por dificultad para la
deglución, niega pérdida de peso. Al Examen físico se objetiva leve palidez, coiloniquia, queilosis
angular, usted le indica una endoscopía para el estudio. ¿Cuál de las siguientes será el hallazgo
más probable?
A. Membrana esofágica proximal C. Pseudoacalasia
B. Membrana esofágica distal D. Adenocarcinoma distal

52. Mujer de 25 años quien padece de ardor retroesternal con episodios de regurgitación e
hipersalivación. ¿Cuál es el diagnóstico probable?
A. Acalasia C. Divertículo de Zenker
B. ERGE D. Adenocarcinomade esófago

53. Mujer de 40 años, con colelitiasis en lista de espera de cirugía, es traída a Emergencia por dolor
abdominal intenso en mesogastrio, coloración amarillenta de escleras, naúseas y vómitos, los
familiares refieren que hace cuatro días, se le administró analgésicos con mejoría parcial. Al
examen: PA 120/75 mmHg, FC 86 x´, T 37.8° C, abdomen doloroso en epigastrio, amilasa en 400,
creatinina 0.6mg/dL. ¿Qué examen es mejor para descartar complicaciones de la patología en
sospecha?
A. Ecografía de hígado y vía biliar
B. Tomografía Abdominal sin y con contraste
C. RMN abdominal
D. CTPH

54. Varón de 57 años acude a Emergencia por heces negras, mal olientes, niega vómitos, niega
pérdida de peso, como antecedente refiere tomar analgésicos durante 5 días por automedicación
. Al examen: palidez leve, PA 120/75mmHg, FC 78 x´, FR 21x´, Sat 98% sin O2 complementario,
abdomen no doloroso. ¿Cuál es el diagnostico más probable?
A. Úlcera gástrica C. NM gástrico
B. Úlcera duodenal D. NM duodenal

55. De los siguientes enunciados. ¿Cuál se relaciona con cuadro de colestasis?


A. Elevación de bilirrubinas a predominio indirecto
B. Valores normales de FA y GGT
C. Elevación marcada de AST y ALT
D. Elevación de bilirrubinas predominio directo, FA y GGT

56. Son factores de riesgo para cáncer gástrico, excepto:


A. Postgastrectomía por úlcera péptica C. Anemia perniciosa
B. Helicobacter Pylori D. Pólipo glandular fúndico

57. El marcador más importante para el diagnóstico de anemia carencial crónica es …


A. Ferritina C. Hemosiderina
B. Reticulocitos D. Transferrina

12 de noviembre del 2022


Página 8 de 21
Examen tipo ENAM MDSURG Academy

58. Varón de 62 años, con cardiopatía isquémica, acude por dolor en región precordial irradiado a
región cervical. Al examen físico: taquicárdico, palidez ++/+++. Laboratorio: HB: 8 g/dL, glucosa:
80mg/dL, creatinina normal. ¿Cuál es el manejo ideal?
A. Transfusión de paquete globular C. Hierro oral
B. Indicar fierro parenteral D.Observación y repetir hemoglobina

59. Varón de 56 años antecedente de gastrectomía por NM gástrico. ¿Cuál es la anemia que puede
presentar a largo plazo?
A. Anemia megaloblástica C. Anemia hemolítica
B. Anemia por deficiencia de folatos D. Anemia por enfermedad crónica

60. Varón de 78 años con antecedente dolor lumbar y de extremidades al deambular además de
astenia de cinco meses de evolución. Al examen: PA: 140/80 mmHg, FC:78 lat/minuto, FR: 20 por
minuto. Palidez ++/+++, hepatoesplenomegalia. Laboratorio: anemia, hipercalcemia, azoemia.
¿Cuál es el diagnóstico más probable?
A. Leucemia mieloide crónica C. Mieloma múltiple
B. Leucemia de células plasmáticas D. Leucemia linfática crónica

61. Mujer de 30 años, soltera; hace 2 años falleció su madre, desde entonces tiene temor de utilizar
el ascensor y evita estar sola. Hace 30 minutos presenta palpitaciones, miedo intenso, temor a
una desgracia inminente y a morir. Al examen: sudorosa con temblor en las manos, inestabilidad
para mantenerse de pie. ¿Cuál es el diagnóstico probable?
A. Crisis de angustia C. Estrés postraumático
B. Depresión mayor D. Esquizofrenia

62. Mujer de 25 años, secretaria refiere que le han puesto micrófonos en su oficina y la están
siguiendo porque el gobierno piensa que es una espía. ¿Cuál es el diagnóstico más probable?
A. Ansiedad generalizada C. Esquizofrenia paranoide
B. Trastorno obsesivo compulsivo D. Trastorno bipolar

63. Mujer que se realiza cortes en piernas y antebrazos cuando se pelea con su enamorado, además
es impulsiva. ¿Qué tipo de personalidad tiene?
A. Histriónica C. Bipolar
B. Personalidad limítrofe D. Personalidad obsesivo compulsiva

64. Escolar de 9 años presenta lesiones pruriginosas en cuero cabelludo. Al examen: lesiones
papulares que se extiende en forma concéntrica formando placas eritematosas, con
descamación central con pelo frágil y algunas áreas de alopecia. ¿Cuál es el diagnóstico más
probable?
A. Alopecia areata C. Psoriasis punctata
B. Tiña capitis D. Tricotilomanía

65. Varón de 22 años con pápulas en frente y nariz que remiten espontáneamente. ¿Cuál es el
diagnóstico?
A. Acné Vulgar C. Impétigo
B. Rosácea D. Lupus discoide

66. Varón de 66 años desde hace 2 meses presenta lesión papular eritematosa de 1 cm en dorso,
hace 1 semana se ulceró con leve sangrado. ¿Cuál es el diagnóstico probable?
A. Hemangioma C. Leishmaniosis cutánea
B. Herpes simple D. Carcinoma basocelular

67. Paciente mujer de 51 años, procedente de Motupe, con antecedente de cálculos en la vesícula
biliar hace 15 años, refiere episodios de dolor en HCD, tratado con inyecciones EV en Emergencia
de su centro médico, no fue operada. Hace 2 días inicia con cuadro de distensión abdominal,
vómitos biliosos y cólico abdominal moderado, la principal sospecha sería:
A. Bridas y adherencias C. NM intestinal
B. Hernia inguinal silente D. Íleo biliar

12 de noviembre del 2022


Página 9 de 21
Examen tipo ENAM MDSURG Academy

68. Un paciente en espera de cirugía de vesícula y que desarrolla coledocolitiasis. ¿Cuál es la


conducta a seguir?
A. Exploración de vía viliar + Kher C. CPRE, después Colelap
B. Colelap y después CPRE D. Control en 1 mes

69. Las hernias femorales, según la clasificación de Nyhus es:


A. Tipo I C. Tipo III
B. Tipo II D. Tipo IV

70. Paciente procedente de la zona sur oriental del Perú con clínica de distensión abdominal
marcada, vómitos fecaloides, ausencia de eliminación de flatos. ¿La principal sospecha será?:
A. Estenosis rectal maligna C. Obstrucción de intestino delgado
B. Vólvulo de sigmoides D. Hemorroides estrangulada

71. Se encuentra en la Emergencia, junto a su asistente de cirugía, quien describe en el paciente el


signo de Dunphy positivo, en qué patología se encuentra presente.
A. Colecistitis aguda C. Pancreatitis aguda
B. Diverticulitis aguda D. Apendicitis aguda

72. Paciente varón de 42 años, antecedente de COLELAP de emergencia reciente, dado de alta al
PO2, reingresa por Emergencia al PO6 refiriendo dolor moderado en HCD, fiebre de 38.9 ºC, herida
operatoria sin flogosis. La principal sospecha es:
A. Neumonía basal C. Atelectasia
B. Absceso residual D. Perforación intestinal

73. ¿Cuál de las siguientes caracteriza a la técnica de LICHTENSTEIN?


A. Sin tensión, abordaje anterior, malla reabsorbible
B. Sin tensión, abordaje posterior, malla irreabsorbible
C. Con tensión, abordaje anterior, malla irreabsorbible
D. Sin tensión, abordaje anterior, malla irreabsorbible

74. Paciente mujer de 75 años, antecedente de estreñimiento sin otras patologías. Refiere un tiempo
de enfermedad de 3 días, con dolor en fosa ilíaca izquierda, moderada intensidad, no irradiado,
asociado a hiporexia, náuseas y malestar general. Al examen físico destaca taquicardia de 110
lat/min, Febril 38.3 ºC, y dolor a la palpación en FII. De las siguientes medidas es incorrecta.
A. Soliticar una TEM abdominal pélvica con contraste, si creatinina normal.
B. Se sospecha de una diverticulitis aguda
C. Solicitar una colonoscopía
D. Descartar un abdomen agudo quirúrgico

75. Paciente varón de 35 años, antecedente de COLELAP hace 1 año, inicia con dolor abdominal en
hipocondrio derecho e ictericia y fiebre en picos. Su principal sospecha será:
A. Pancreatitis aguda C. Lesión de vía biliar
B. Colangitis aguda por coledocolitiasis residual D. Colelitiasis

76. El signo radiológico que sugiere íleo biliar es:


A. Asa centínela en fosa ilíaca izquierda C. Imagen en " cuentas de rosario"
B. Signo de "tiro al blanco" en FID D. Aerobilia o neumobilia

77. Durante su SERUMS, recibe en consulta a paciente con molestias perianales, en el examen físico
encuentra una hemorroide centinela, dicho hallazgo es característico de:
A. Hemorroides externas C. Hemorroides internas grado II
B. Absceso perianal D. Fisura anal crónica

78. En cirugía gástrica. El grupo ganglionar Nº5, corresponde a los ubicados en:
A. Paracardial derecho C. Curvatura menor
B. Hilio esplénico D. Suprapilóricos

12 de noviembre del 2022


Página 10 de 21
Examen tipo ENAM MDSURG Academy

79. Niña de 4 años que es llevado a Emergencia por dolor y limitación funcional del codo derecho.
Examen físico: pronación dolorosa. ¿Cuál es el diagnóstico más probable?
A. Fractura de codo C. Fractura de Colles
B. Codo de niñera D. Fractura de Smith

80. Paciente varón de 35 años, trabajador de fábrica de vidrio, acude porque accidentalmente sufre
corte en región de antebrazo, al examen se evidencia mano en garra, este hallazgo se produce
por lesión del:
A. Radial C. Mediano
B. Cubital D. Axilar

81. Paciente de sexo femenino de 45 años, secretaria de una Universidad. A la exploración: Signo de
Tinel. ¿Cuál es el diagnóstico?
A. Neuropatía diabética C. Síndrome de túnel carpiano
B. Mononeuritis múltiple D. Parálisis del plexo braquial

82. Un niño de 4 meses de edad presenta signo de Barlow y al examen presenta limitación a la
abducción de la cadera izquierda. La conducta más adecuada es:
A. Solicitar radiografía AP de pelvis
B. Resolver quirúrgicamente de inmediato
C. Solicitar radiografía de la cadera izquierda
D. Solicitar ecografía de cadera

83. Mujer de 35 años, viajaba como pasajera en la parte posterior del auto que sufrió una colisión
frontal. Ingresa al servicio de Emergencia en decúbito dorsal, con intenso dolor en la cadera
derecha. Al examen: rotación interna y aducción del miembro inferior ipsilateral. ¿Cuál es el
diagnóstico?
A. Luxación anterior de cadera
B. Luxación de rodilla y tobillo
C. Fractura de Fémur subcapital
D. Luxación posterior de cadera

84. Varón de 30 años llevado a Emergencia por puñalada en cara lateral de hemitórax derecho.
Examen: PA: 110/70 mmHg y FC: 65 X’. Respiratorio: desviación traqueal hacia la izquierda con
murmullo vesicular abolido en hemitórax derecho. ¿Cuál es el diagnóstico clínico?
A. Hemotórax derecho C. Neumotórax derecho
B. Mediastinitis D. Hemotórax izquierdo

85. ¿Cuál es el volumen de sangre a través del tubo de drenaje que decide una toracotomía en un
paciente con trauma torácico?
A. 750ml C. 2000 ml
B. 1500ml D. 2500 ml

86. Varón de 28 años, con tiempo de enfermedad de 6 horas, presenta disnea progresiva. Examen
físico: ingurgitación yugular, presión arterial 90/60 mmHg, tráquea desviada hacia el lado
derecho e hipersonoridad a la percusión del hemitórax izquierdo. ¿Cuál es el manejo quirúrgico
de emergencia?
A. Descompresión con aguja C. Toracoscopía
B. Tubo de Drenaje torácico D.Toracotomía de emergencia

87. ¿Qué manifestación clínica obliga a la resolución quirúrgica de un recién nacido con
enterocolitis necrotizante?
A. Hematoquezia C. Vómitos biliosos
B. Neumoperitoneo D. Shock hipovólemico

12 de noviembre del 2022


Página 11 de 21
Examen tipo ENAM MDSURG Academy

88. Neonato de 25 días presenta desde hace 4 días vómitos lácteos a chorro inmediatamente
después de lactar, que se hacen más frecuentes cada día. ¿Cuál es el trastorno metabólico que
se espera encontrar?
A. Alcalosis hipoclorémica C. Alcalosis respiratoria
B. Acidosis hipoclorémica D. Alcalosis digestiva

89. Lactante de 18 meses, es traído a Emergencia por presentar crisis de llanto incontrolable en
forma súbita a intervalos frecuentes, con flexión de las piernas hacia el abdomen y se evidencia
deposición con sangre y moco, masa palpable en abdomen. Niega antecedentes patológicos.
¿Cuál es el diagnóstico?
A. Diarrea disénterica C. Estenosis duodenal
B. Invaginación intestinal D. Páncreas anular

90. En pacientes con antecedente de cólicos renales frecuentes asociados a litiasis renal. ¿Cuál es
la bacteria que se asocia con mayor frecuencia?
A. Escherichia Coli C. Estafilococo saprophyticus
B. Enterococos faecalis D. Proteus mirabilis

91. ¿Cuál es el tratamiento quirúrgico de emergencia de la ruptura de uretra posterior?


A. Uretrotomía C.Talla vesical
B. Cateterismo vesical D.Uretroplastía T-T

92. Varón de 15 años refiere dolor testicular derecho de 6 horas de evolución. Antecedente de
testículo retráctil del mismo lado. Al examen físico destaca discreta horizontalización de
testículo derecho y reflejo cremastérico ausente. ¿Cuál es el diagnóstico más probable?
A. Epididimitis aguda C. Torsión testicular
B. Hidrocele testícular D. Hernia inguino-escrotal

93. Varón de 30 años sufre quemadura con agua hervida. Examen: lesión que compromete ambas
caras anteriores de las extremidades inferiores. Según la regla de los nueves, ¿Cuál es la
extensión de la quemadura?
A. 9% C. 27%
B. 18% D. 36%

94. Paciente varón de 52 años, ingresa a Emergencia y refiere que sufrió mordedura de perro con
vacunas completas, mordedura de 2.5 cm en miembro inferior derecho hace 6 horas, usted
realiza curación exhaustiva y le administra toxoide antitetánico. Le explica que no requiere
sutura, le indica antibiótico y analgésico con cita en consultorio externo en 5 días. El tipo de
cicatrización presente en el siguiente caso es:
A. Primera intención C. Segunda intención
B. Tercera intención D. Primaria tardía

95. ¿Cuál de los siguientes signos o síntomas clínicos proporciona el diagnóstico más preciso de
glaucoma agudo?
A. Congestión conjuntival C. Miosis
B. Midriasis D. Dolor ocular moderado

96. De las siguientes alternativas. ¿Cuál es el síntoma que se presente en el desprendimiento de


retina después de un traumatismo?
A. Percepción de destellos de luz C. Midriasis
B. Visión tubular D. Miosis

97. ¿Cuál es el germen causante de la otitis externa maligna?


A. Klebsiella neumoniae C. Pseudomona aeruginosa
B. Streptococcus beta hemolítico D. Streptococcus pneumoniae

12 de noviembre del 2022


Página 12 de 21
Examen tipo ENAM MDSURG Academy

98. ¿Cuál es la causa más común de hemorragia nasal por lesión del plexo de Kiesselbach?
A. Traumatismo nasal C. Rinitis
B. AAS D. Desviación del tabique nasal

99. Varón de 32 años, antecedente de tránsito en motorizado lineal con contusión frontal izquierda
hace 4 semanas, TAC de primera atención sin lesiones evidentes. Reingresa por cefalea intensa,
náuseas y mareos. Al examen: alteración de la conciencia, escala de Glasgow 13/15. TAC
cerebral: imagen hipodensa en forma de media luna. ¿Cuál es su diagnóstico probable?
A. Hematoma subdural crónico C. Hemorragia subaracnoidea
B. Hematoma epidural D. Hemorragia intraparenquimatosa

100. Paciente, chofer de carga pesada, se presenta con dolor lumbar irradiado a la pierna izquierda.
El dolor recorre la cara anterior y medial del muslo y al examen físico usted encuentra debilidad
a la extensión de la rodilla y disminución en la intensidad del reflejo rotuliano. La raíz nerviosa
comprometida es:
A. L4 C. L5
B. L3 D. L1

101. Ingresa paciente varón de 45 años a la unidad de Shock trauma con diagnóstico de muerte
cerebral, familia decide donación de órganos, el tipo de ASA es:
A. ASA III C. ASA V
B. ASA IV D. ASA VI

102. ¿Qué medicamento se usa como relajante muscular en la técnica de inducción-intubación en


secuencia rápida?
A. Succinilcolina C. Etodimato
B. Rocuronio D. Ketamina

103. Adolescente de sexo femenino que presenta aumento de la mama y la areola, no hay
separación del contorno. ¿A qué estadío de Tanner corresponde?
A. M5 C. M1
B. M4 D. M3

104. Niño de 11 meses de edad al ser evaluado se observa que sostiene la cabeza, permanece
sentado en posición de trípode, coge objetos con toda la palma, pero aún no se mantiene en
pie. ¿Cuál es el diagnóstico?
A. Retardo moderado del desarrollo C. Retardo severo del desarrollo
B. Desarrollo normal D. Leve retardo

105. Lactante que rueda de prono a supino y se sienta sin soporte. ¿Cuál es su edad en meses?
A. 3 C. 4
B. 5 D. 6

106. Niño que tiene vacunación sólo hasta el año de edad. ¿Cuál de las vacunas está completa?
A. Contra el neumococo C. DPT
B. SPR D. Contra la influenza

107. Lactante varón de 4 meses de edad, con un peso de 5,5 Kg y talla de 51 cm. Si en el percentil
50 el peso/edad es de 6.8 y la talla es de 63 cm y el peso/talla de 6.5 Kg. ¿Cuál es su estado
nutricional?
A. Desnutrición crónica compensada
B. Desnutrición aguda
C. Desnutrición crónica descompensada
D. Eutrófico

12 de noviembre del 2022


Página 13 de 21
Examen tipo ENAM MDSURG Academy

108. Niña de 5 años con talla baja para la edad, piel cabello quebradizo, presenta edema
generalizado. ¿Diagnóstico nutricional más probable?
A. Marasmo C. Enanismo
B. Kwashiorkor D. Déficit de vitamina A

109. Niño de 2 años evaluado por presentar cianosis al llanto. Al examen radiológico: Corazón de
tamaño normal, elevación de la punta y concavidad en la región de la arteria pulmonar. ¿Cuál
es la sospecha diagnóstica?
A. Tetralogía de Fallot
B. Atresia tricúspidea
C. Retorno venoso pulmonar anómalo completo
D. Transposición de grandes vasos

110. Paciente de 4 años, es traído a consulta por presentar lesiones dérmicas pruriginosas. Al
examen: lesiones vesiculares en cuero cabelludo y cuello, algunas lesiones costrosas en tórax,
microadenopatía generalizada, orofaringe congestiva. ¿Cuál es el diagnóstico?
A. Rubéola C. Varicela
B. Escarlatina D. Prurigo

111. Lactante de 10 meses con febrícula de 3 días, presenta enrojecimiento facial que se extiende
al tronco y extremidades proximales respetando palmas y plantas. ¿Cuál es el diagnóstico más
probable?
A. Escarlatina C. Enfermedad mano-pie-boca
B. Rubéola D. Eritema infeccioso

112. Escolar de 8 años con cuadro febril de 3 días de evolución, es traído por erupción puntiforme
generalizada. Al examen: FC: 98 lat/min. T: 38ºC FR: 22 resp/min. Piel: con erupción pápulo -
eritematosa generalizada, edema facial que respeta el surco nasogeniano, lengua en fresa,
orofaringe congestiva y amígdalas hipertróficas. ¿Cuál es el diagnóstico más probable?
A. Sarampión C. Escarlatina
B. Kawasaki D. Roséola

113. Paciente de 3 años de edad, P: 12 kg; desde hace 2 días presenta deposiciones líquidas acuosas
10 cámaras, fiebre, hiporexia, vómitos post-prandiales. Al examen: T 38,5°C, F.C.: 100 lat/min,
piel y mucosas secas, eutrófico, abdomen no distendido, RC rítmicos y de buena intensidad. El
manejo inicial es:
A. Iniciar antibioticoterapia
B. Fluidoterapia con solución lactato de Ringer
C. Rehidratación oral
D. Fluidoterapia EV con solución polielectrolítica

114. Preescolar de 4 años presenta desde hace 3 días fiebre y deposiciones líquidas en número de
4 a 6 por día, inicialmente sin moco ni sangre, de escaso volumen, posteriormente con moco y
rasgos de sangre, acompañados de dolor abdominal y tenesmo. El agente etiológico presuntivo
es:
A. Salmonella typhi C. Rotavirus
B. Vibrio cholerae D. Shigella

115. Paciente de 1 año presenta desde hace 4 días deposiciones líquidas con moco, sin sangre,
vómitos y fiebre. Desde hace 1 día los vómitos se han intensificado, presenta deposiciones con
sangre, desarrolla palidez marcada y oliguria. El diagnóstico es:
A. Síndrome urémico hemolítico C. Disentería amebiana
B. Disentería por Shigella D. Intususcepción

12 de noviembre del 2022


Página 14 de 21
Examen tipo ENAM MDSURG Academy

116. Paciente de 18 meses es traída a la Emergencia por presentar una convulsión tónico-clónico en
casa que se desarrolló mientras el paciente presentó fiebre, dicha convulsión duró 5 minutos.
Ahora en la puerta de Emergencia se evidencia otra convulsión de características similares.
¿Cuál es el diagnóstico más probable?
A. Epilepsia C. Convulsión febril compleja
B. Convulsión febril simple D. MEC

117. Neonato que presenta una tumoración a nivel lumbar sin secreción de LCR. ¿Cuál es el
diagnóstico?
A. Mielocele C. Hemimielocele
B. Teratoma D. Mielomeningocele

118. La cardiopatía más frecuente asociada a la prematuridad es:


A. Comunicación interauricular C. Tetralogía de Fallot
B. Coartación de aorta D. Persistencia del ductus arterioso

119. Recién nacido con líquido meconial espeso, presenta llanto vigoroso, con buen esfuerzo
respiratorio. ¿Qué actitud se debe tomar?
A. Aspirar la boca, luego la faringe posterior, hipofaringe y la nariz
B. Secado - aspiración - observación
C. Intubación
D. Aspiración de secrecione

120. Recién nacido de 2 días, es llevado a la EMG por presentar hipoactividad, pobre succión, con
antecedente de madre sin control prenatal y RPM de 20 horas. ¿Cuál es el diagnóstico?
A. Sepsis neonatal tardía C. Asfixia perinatal
B. Sepsis neonatal temprana D. Encefalopatía Hipóxico-isquémica

121. Recién nacido de 38 semanas por cesárea presenta dificultad respiratoria, quejido inconstante,
tiraje subcostal y aleteo nasal leve, retracción xifoidea. Ruidos cardíacos normales. ¿Cuál es el
diagnóstico más probable?
A. Membrana hialina C. Taquipnea transitoria
B. Aspiración meconial D. Neumonía neonatal

122. Recién nacido, presenta distrés respiratorio marcado, timpanismo en hemitórax izquierdo, ruídos
cardíacos desplazados a hemitórax derecho. Murmullo vesicular pasa bien en hemitórax derecho.
¿Cuál es su diagnóstico?
A. Hernia diafragmática C. Hidroneumotórax
B. Hemotórax D. Neumotórax

123. Lactante de 22 días de nacido, con bajo peso al nacer, presenta sangre en heces y neumatosis
intestinal en la radiografía simple de abdomen. ¿Qué diagnóstico es el más probable?
A. Íleo meconial C. Enterocolitis necrotizante neonatal
B. Hernia inguinal D. Absceso de psoas

124. Recién nacido que presenta vómitos biliosos y distensión abdominal progresiva. ¿Cuál es el
diagnóstico más probable?
A. Hipertrofia pilórica C. Íleo por meconio
B. Vólvulo de intestino medio D. Atresia intestinal

125. Madre trae a su hijo recién nacido de 6 días por presentar coloración amarillenta en la piel desde
hace 48 horas. Al examen físico: se encuentra ictericia generalizada. Examen de laboratorio:
Bilirrubina total 12 mg/dL a predominio indirecto. ¿Cuál es el diagnóstico?
A. Ictericia por lactancia materna
B. Ictericia fisiológica
C. Enfermedad hemolítica del recién nacido
D. Incompatibilidad ABO

12 de noviembre del 2022


Página 15 de 21
Examen tipo ENAM MDSURG Academy

126. RN de 20 horas de vida que presenta hipoactividad, polipnea y plétora. EG de 42 semanas por
Capurro. Madre con glucosa 55 mg/dL. ¿Cuál es el diagnóstico más probable?
A. Hipoglicemia C. Policitemia
B. Hiponatremia D. Sepsis neonatal

127. Neonato de 6 días de madre primigesta, sin control prenatal fue atendida en su domicilio. Es
hospitalizado por presentar irritabilidad, dificultad para succionar, espasmos musculares ante
diferentes estímulos. Al examen: febril, despierto en hiperextensión corporal, con aducción de los
brazos e hiperextensión del cuello. ¿Cuál es el diagnóstico?
A. Hipoglicemia C. Tétanos neonatal
B. Sepsis neonatal D. Meningitis neonatal

128. Niño nacido de parto normal hace 2 días. Desde hoy presenta edema palpebral y abundante
secreción purulenta en ambos ojos. El diagnóstico más probable es conjuntivitis neonatal por:
A. Herpes simple C. Estreptococo
B. Clamidia D. Gonococo

129. Recién nacido pretérmino de 35 semanas con diagnóstico de hipoglicemia. ¿Cuál de los siguientes
hallazgos es menos probable encontrar?
A. Convulsiones C. Pobre succión
B. Irritabilidad D. Hipertermia

130. Recién nacido de 14 horas de madre primípara de 18 años, quién a las 32 semanas presentó
infección urinaria gestacional desencadenando el parto. Al examen: Peso al nacer 1,400 gr. y test
de Silverman Anderson 8. Hemograma: normal. ¿Cuál es su diagnóstico?
A. Membrana hialina C. Síndrome de aspiración
B. Taquipnea transitoria D. Cardiopatía congénita

131. Gestante de 41 años multípara, con antecedente de hipertensión crónica, actualmente en 32


semanas de gestación con PA 150/110 mmHg y proteinuria de 300 mg/24h. ¿Qué antihipertensivo
es el indicado?
A. Atenolol C. Alfa metildopa
B. Captopril D. Losartán

132. ¿Cuál de las siguientes presentaciones NO tiene opción de parto vaginal?


A. Bregma C. Occipucio
B. Cara D. Frente

133. Una gestante de 30 semanas inicia con metrorragia de moderada cantidad de color rojo brillante,
sin otras molestias. No presenta contracciones con pruebas de bienestar fetal sin alteraciones. El
diagnóstico más probable es:
A. Rotura de vasa previa
B. Placenta previa
C. DPP
D. Desprendimiento de placenta normoinserta

134. Una mujer de 28 años, embarazo de 9 semanas, confirmado con test de detección de beta-HCG
urinaria. Acude a Emergencia por dolor hipogástrico intenso y sangrado vaginal. En la Emergencia
presenta lipotimia y ortostatismo. Al examen físico se aprecia pálida, con frialdad de
extremidades, FC: 124x’ y PA: 80/50 mmHg. El diagnóstico de sospecha es:
A. DPP C. Aborto incompleto
B. Embarazo molar D. Embarazo ectópico roto

12 de noviembre del 2022


Página 16 de 21
Examen tipo ENAM MDSURG Academy

135. Son parámetros evaluados en el Perfil Biofísico Fetal, Excepto:


A. Movimientos fetales y respiratorios
B. Volumen de líquido amniótico
C. Tono fetal
D. Posición fetal

136. Paciente de 25 años con 22 semanas de gestación, se presenta a Emergencia con dolor
abdominal y sangrado vaginal escaso de 48 horas de evolución. Al examen: altura uterina 20 cm,
movimientos fetales presentes. Especuloscopía: se observa membranas ovulares prominentes e
íntegras, que protruyen por un orificio externo abierto. ¿Cuál es el diagnóstico?:
A. Aborto inminente C. Aborto frustro
B. Aborto incompleto D. Aborto inevitable

137. Mujer de 36 años, gestante de 31 semanas, acude a consulta por cefalea y malestar general. Al
examen físico presenta: PA 150/100 mmHg, FC 105 x´, edema de extremidades inferiores y ROT
exaltados. ¿Cuál es el diagnóstico más probable?
A. HTA crónica C. Preeclampsia leve
B. Síndrome de HELLP D. Preeclamsia severa

138. Se denomina embarazo prolongado postérmino cuando este excede la semana:


A. 40 C. 42
B. 41 D. 44

139. Maniobra obstétrica que consiste en la flexión intensa de los muslos de la madre sobre su
abdomen y que se puede emplear en la distocia de hombros en parto podálico, se conoce como:
A. Benedeti C. Bracht
B. Mc Roberts D. Woods

140. Gestante de 37 semanas, con VIH confirmado, sin labor de parto. ¿Cuál sería la vía de culminación
de embarazo más adecuada?
A. Parto vaginal C. Cesárea de urgencia
B. Parto instrumentado D. Cesárea programada

141. Primigesta de 30 años, con 38 semanas de gestación y control prenatal irregular. Hace 48 horas
inicia con cefalea, visión borrosa y dolor en epigastrio. Al examen PA 180/110 mmHg, FC 112x´,
ictericia leve, leucocitosis leve, hemoglobina en 9.1 g/dL, Plaquetas 35.000; TGO y TGP elevados,
BT a predominio indirecto elevados, creatinina 1.4. Examen de orina con proteínas ++++,
hematuria. ¿Cuál es la medida más adecuada?
A. Terminar la gestación de inmediato
B. Interconsulta UCI y observar evolución
C. Interconsulta a Cirugía y transfundir plaquetas
D. Iniciar hipotensores e inducción de parto

142. En una presentación de vértice, si la sutura sagital es transversa u oblicua, pero está más cercana
a la sínfisis del pubis que al promontorio, se presenta una condición específica. ¿Cómo se
denomina a ésta?
A. Asinclitismo anterior C. Rotación interna
B. Asinclitismo posterior D. Rotación externa

143. Mujer de 32 años, sufre una laceración de perineo durante el parto que afecta al esfínter anal sin
afectación de la mucosa rectal. Esta laceración puede clasificarse como grado:
A. I C. IV
B. II D. III

12 de noviembre del 2022


Página 17 de 21
Examen tipo ENAM MDSURG Academy

144. Mujer de 35 años, gestante de 38 semanas, primigesta, con feto en presentación pélvica, sin labor
de parto .¿Cuál es la conducta obstétrica más adecuada?
A. Cesárea de emergencia C. Cesárea programada
B. Inducción de trabajo de parto D. Esperar el parto vaginal

145. Mujer de 30 años, con embarazo de 35 semanas, acude a Emergencia por salida de líquido vaginal
escaso, filante, con escasas estrías de sangre. Perfil biofísico fetal sin alteraciones. ¿Cuál es el
diagnóstico más probable?
A. Rotura prematura de membranas C. Eliminación de tapón mucoso
B. Rotura de vasa previa D. Placenta previa

146. Mujer de 40 años con embarazo de 35 semanas por ecografía, presenta sangrado oscuro,
asociado a contracciones uterinas dolorosas. Al examen físico se encuentra hipertonía uterina y
bradicardia fetal. ¿Cuál es el diagnóstico más probable?
A. Rotura uterina
B. Placenta previa oclusiva
C. Rotura de Vasa previa
D. Desprendimiento prematuro de placenta normoinserta

147. Paciente de 36 años, en sus 31 semanas de gestación con rotura prematura de membranas. ¿Cuál
es la conducta que recomendaría en esta paciente?
A. Terminar el embarazo por la vía más rápida
B. Dejar evolucionar espontáneamente
C. Administrar antibióticos, corticoides, con conducta expectante
D. Terminar el embarazo con cobertura antibiótica

148. Puérpera de 40 años con parto eutócico y retención de membranas amnióticas. ¿Cuál es la
conducta a seguir?
A. Legrado uterino C. Extraer con pinza y gasa
B. Administrar oxitocina D. Control por consultorio

149. El aumento de peso recomendado durante el embarazo en una mujer con IMC normal es:
A. 9.5 a 11 kg C. 5 A 7 Kg
B. 16.5 a 18 kg D. 11.5 a 16 Kg

150. Gestante de 38 semanas que acude a Emergencia por presentar ruptura prematura de
membranas. Se decide inducir el parto. ¿Cuál es el fármaco recomendado?
A. Ergotamina C. Oxitocina
B. Metilergonovina D. Ergonovina

151. Son factores de riesgo para rotura prematura de membranas, EXCEPTO:


A. Primigesta C. Polihidramnios
B. Embarazo gemelar D. Parto prematuro previo

152. El aumento del grosor de la translucencia nucal es un hallazgo que se asocia a:


A. Defectos del tubo neural C. Isoinmunización Rh
B. Síndrome de Down D. Rubéola

153. Una mujer presenta abundante sangrado vaginal después del alumbramiento. Al examen físico
se palpa fondo de útero entre la apófisis xifoides y el ombligo. ¿Cuál es el diagnóstico más
probable?
A. Laceración de canal de parto C. Atonía uterina
B. Rotura uterina D. Acretismo placentario

12 de noviembre del 2022


Página 18 de 21
Examen tipo ENAM MDSURG Academy

154. Adolescente de 16 años, hace cinco días ha notado secreción sanguinolenta espontánea por el
pezón de la mama derecha. Sin antecedentes familiares de patología mamaria. ¿Cuál es el
diagnóstico probable?
A. Tumor phyllodes de la mama C. Mastopatía fibroquística de mama
B. Papiloma intraductal D. Fibroma de mama

155. Mujer de 20 años, sexualmente activa acude a Emergencia con dolor pélvico, flujo con mal olor.
Al examen: PA: 120/80 mmHg, FC 78 x´, T 37.8ºC, dolor a la movilización cervical, y dolor bilateral
a la palpación en fosa pélvica. ¿Cuál es el diagnóstico más probable?
A. Dismenorrea C. EPI
B. Apendicitis aguda D. Endometriosis

156. Mujer de 23 años sufre violación sexual. Al examen presencia de equimosis en región inguinal
izquierda y erosiones en labios mayores. ¿Cuál sería el anticonceptivo de emergencia?
A. Misoprostol C. Etinilestradiol
B. Aplicación de DIU D. Levonorgestrel

157. ¿Cuál de los siguientes es factor de riesgo para cáncer de mama?


A. Menopausia precoz C. Menarquia temprana
B. Menarquia tardía D. ACO

158. Paciente de 30 años G0 P0, acude a Emergencia refiriendo sangrado vaginal abundante hace una
semana, llegando a utilizar 10 a más paños al día, al examen paciente con palidez ++/+++,
Especuloscopía sangrando activo en abundante cantidad. ¿Cuál es el tipo de mioma que explica
el caso?
A. Subseroso C. Intramural
B. Submucoso D. Pediculado

159. Mujer de 32 años; G0P0, FUR: hace 5 días, R.C. 5/28, antecedente de dismenorrea y dispareunia;
con dolor pélvico intenso. Examen: FV: normales, abdomen blando con dolor a la palpación
profunda en hipogastrio, sin rebote. TV: tumor anexial derecho. Laboratorio: normal. Ecografía:
masa anexial unilocular derecha de 5 cm, con “apariencia de vidrio esmerilado”. ¿Cuál es el
diagnóstico más probable?
A. Folículo hemorrágico C. Endometriosis
B. Embarazo ectópico D. Quiste a pedículo torcido

160. Mujer de 21 años, sexualmente activa, con prurito vaginal intenso y flujo vaginal. Examen
ginecológico: secreción verdosa en el canal vaginal y cérvix con punteado rojizo, con apariencia
de "fresa". ¿Cuál es el tratamiento indicado?
A. Gentamicina IM C. Metronidazol oral
B. Metronidazol tópico D. Clotrimazol tópico

161. El contar con un servicio de emergencia con especialistas capacitados debe ser considerado:
A. Fortaleza C. Estrategia
B. Oportunidad D. Amenaza

162. ¿A qué nivel de prevención corresponde la búsqueda de sintomático respiratorio?


A. Primaria C. Terciaria
B. Secundaria D. Primordial

163. Según la norma técnica de salud para el control de la tuberculosis, se considera como tratamiento
complementario.
A. Nutrición y terapia psicológica C. Actividad física
B. Aislamiento D. Tratamiento alternativo

12 de noviembre del 2022


Página 19 de 21
Examen tipo ENAM MDSURG Academy

164. Acude a emergencia paciente con dolor lumbar moderado post esfuerzo moderado, cede
parcialmente a la medicación. El médico le indica una Radiografía dorsolumbar, el paciente no da
consentimiento para la toma de dicha imagen. Este hecho del paciente se basa en el principio de
A. Libertad C. Autonomía
B. Justicia propia D. No maleficencia

165. Según la Norma Técnica 021-MINSA, de acuerdo con la categorización da establecimientos de


salud, el centro de salud que cuenta con internamiento pertenece al nivel:
A. I-2 C. I-4
B. I-3 D. II-1

166. Personaje público llega en mal estado a Emergencia, por intento de suicidio, un personal de salud
le toma una fotografía y lo pública en redes sociales. ¿Qué principio ético se ha vulnerado
principalmente en este caso?
A. Autonomía C. Justicia
B. Beneficencia D. No maleficencia

167. Se calcula el IMC de 80 pacientes diabéticos y de 150 pacientes sanos. ¿Qué prueba estadística
es la más adecuada para determinar si existen diferencias significativas entre ambos grupos?
A. t de Student C. Chi- cuadrado
B. Análisis de Varianza D. r de Pearson

168. Cuando en una investigación se asume erróneamente que la hipótesis nula es falsa. ¿Qué tipo de
error se está cometiendo?
A. II (beta) C. I (alfa)
B. De aleatorización D. De diseño metodológico

169. Cómo se estima el Riesgo relativo (RR) y Odds Ratio (OR):


A. La fracción etiológica o atribuible a la exposición
B. La proporción de enfermos que se debe a la exposición
C. La diferencia de riesgo entre expuestos y no expuestos
D. Magnitud entre expuestos y no expuestos

170. Uno de los principales inconvenientes de los estudios de casos y controles es:
A. Su mayor susceptibilidad a incurrir en sesgos respecto a otros estudios epidemiológicos
B. Que no son adecuados para estudiar enfermedades raras
C. Que no permiten determinar la odds ratio (OR)
D. Que suelen ser caros y de larga duración

171. Las arterias que conforman el arco de la curvatura menor del estómago son:
A. Gastroduodenal izquierda más gástrica derecha
B. Pilórica más gástrica izquierda
C. Coronaria estomáquica más gastro-omental derecha
D. Vasos cortos y gastro-epiploica izquierda

172. Las venas hemorroidales superiores desembocan en la vena:


A. Mesentérica superior C. Mesentérica inferior
B. Pudenda D. Ilíaca interna

173. ¿Cuál de los siguientes está mediado por la progesterona?


A. Endometrio proliferativo C. Efecto termogénico
B. Moco cervical delgado, filante D. Desarrollo ductal mamario

174. ¿En que población de pacientes se encuentran las úlceras de Curling?


A. Alcohólicos C. Patología intracraneal
B. Por cáncer D. Con quemaduras

12 de noviembre del 2022


Página 20 de 21
Examen tipo ENAM MDSURG Academy

175. El mecanismo de acción del clopidogrel se basa en:


A. Bloqueo de la enzima fosfodiesterasa
B. Inhibición de la glicoproteína IIb/IIIa
C. Inhibición de la ciclooxigenasa
D. Inhibición del receptor plaquetario para ADP

176. El aliskiren es un fármaco con propiedades:


A. Calcio- antagonistas C. Beta- bloqueantes
B. Inhibidoras de la renina D. Vasodilatadoras

177. ¿En qué lugar se produce la secreción de la colecistoquinina (CCK)?


A. Células S del duodeno C. Células I del duodeno y yeyuno
B. Células G del antro D. Vesícula biliar

178. Señale la asociación incorrecta:


A. Furosemida – hipocalcemia. C. Aliskiren – hiperpotasemia.
B. Captopril – hipopotasemia. D. Hidroclorotiazida – hipercalcemia.

179. ¿Cuál de los siguientes animales transmite la toxoplasmosis a la especie humana?


A. Peces C. Gatos
B. Aves D. perros

180. ¿Cuál de los siguientes procesos esta causado por Clostridium perfringens?
A. Enteritis necrosante C. Colitis pseudomembranosa
B. Ectima gangrenoso D. Colitis hemorrágica

12 de noviembre del 2022


Página 21 de 21
21/11/23, 21:14 POST TEST . HDB + SD OBSTRUCCION INTESTINAL .: Revisión del intento

Comenzado el martes, 21 de noviembre de 2023, 21:10


Estado Finalizado
Finalizado en martes, 21 de noviembre de 2023, 21:14
Tiempo empleado 4 minutos 31 segundos
Calificación 20,00 de 20,00 (100%)

Pregunta 1

Correcta

Se puntúa 5,00 sobre 5,00

Varon procedente del callao , llega por dolor abdominal en fosa iliaca izquierda. mas fiebre . deposiciones liquidas ,mas o memnos
4 veces por dia por 3 dias.

diagnostico:

a. GECA

b. hematoquezia

c. todas son corrrectas

d. diverticulitis 
e. angiodisplacia

Respuesta correcta

La respuesta correcta es: diverticulitis

Pregunta 2

Correcta

Se puntúa 5,00 sobre 5,00

Paciente con depocisiones blandas rojo vinosas mas o menos 2 veces por dia hace 3 dias. no se acompaña de dolor abdominal-
diagnostico

a. colitis ulcerativa

b. diverticulitis

c. angiodisplacia de colon 

d. colitis isquemica

e. melena

Respuesta correcta

La respuesta correcta es:


angiodisplacia de colon

https://www.aulauss.edu.pe/mod/quiz/review.php?attempt=3054303&cmid=4472189 1/2
21/11/23, 21:14 POST TEST . HDB + SD OBSTRUCCION INTESTINAL .: Revisión del intento

Pregunta 3

Correcta

Se puntúa 5,00 sobre 5,00

causa mas frecuente de Hemorragia digestiva baja. con sangrado rojo rutilante en las heces.

a. angiodisplacia

b. diverticulitis

c. colitis ulcerativa.

d. hemorroides 

e. melena

Respuesta correcta

La respuesta correcta es:


hemorroides

Pregunta 4

Correcta

Se puntúa 5,00 sobre 5,00

Paciente con iongreso a ala emergecia por dolor abdominal que no cede con aines, asusencia de flatos.

ademas de no defecar hace 3 dias. al examen: ABDOMEN, distendido con RHA: DE 2 A 3 por minuto

en la placa de abdomen: hay imagen en pilas de monedas en 6 asas imntestinales. QUE PARTE DEL INTESTINO ESTA EN LA
IMAGEN DE LA RADIOGRAFIA y esta comprendido obstruccion de la misma.

a. colon ascendente

b. duodeno

c. bridas y adherencias.

d. intestino delgado 

e. Yeyuno

Respuesta correcta

La respuesta correcta es:


intestino delgado

https://www.aulauss.edu.pe/mod/quiz/review.php?attempt=3054303&cmid=4472189 2/2
12/12/23, 20:24 POST TEST . HDB + SD OBSTRUCCION INTESTINAL .: Revisión del intento

Comenzado el martes, 21 de noviembre de 2023, 21:10


Estado Finalizado
Finalizado en martes, 21 de noviembre de 2023, 21:13
Tiempo empleado 3 minutos 7 segundos
Calificación 20,00 de 20,00 (100%)

Pregunta 1

Correcta

Se puntúa 5,00 sobre 5,00

Varon procedente del callao , llega por dolor abdominal en fosa iliaca izquierda. mas fiebre . deposiciones liquidas ,mas o memnos
4 veces por dia por 3 dias.

diagnostico:

a. todas son corrrectas

b. GECA

c. diverticulitis 
d. angiodisplacia

e. hematoquezia

Respuesta correcta

La respuesta correcta es: diverticulitis

Pregunta 2

Correcta

Se puntúa 5,00 sobre 5,00

Paciente con depocisiones blandas rojo vinosas mas o menos 2 veces por dia hace 3 dias. no se acompaña de dolor abdominal-
diagnostico

a. diverticulitis

b. colitis ulcerativa

c. melena

d. angiodisplacia de colon 

e. colitis isquemica

Respuesta correcta

La respuesta correcta es:


angiodisplacia de colon

https://www.aulauss.edu.pe/mod/quiz/review.php?attempt=3054302&cmid=4472189 1/2
12/12/23, 20:24 POST TEST . HDB + SD OBSTRUCCION INTESTINAL .: Revisión del intento

Pregunta 3

Correcta

Se puntúa 5,00 sobre 5,00

causa mas frecuente de Hemorragia digestiva baja. con sangrado rojo rutilante en las heces.

a. diverticulitis

b. hemorroides 

c. melena

d. angiodisplacia

e. colitis ulcerativa.

Respuesta correcta

La respuesta correcta es:


hemorroides

Pregunta 4

Correcta

Se puntúa 5,00 sobre 5,00

Paciente con iongreso a ala emergecia por dolor abdominal que no cede con aines, asusencia de flatos.

ademas de no defecar hace 3 dias. al examen: ABDOMEN, distendido con RHA: DE 2 A 3 por minuto

en la placa de abdomen: hay imagen en pilas de monedas en 6 asas imntestinales. QUE PARTE DEL INTESTINO ESTA EN LA
IMAGEN DE LA RADIOGRAFIA y esta comprendido obstruccion de la misma.

a. duodeno

b. colon ascendente

c. Yeyuno

d. bridas y adherencias.

e. intestino delgado 

Respuesta correcta

La respuesta correcta es:


intestino delgado

https://www.aulauss.edu.pe/mod/quiz/review.php?attempt=3054302&cmid=4472189 2/2
5/12/23, 23:01 post TEST . IOF - STATUS CONVULSIVO.: Revisión del intento

Comenzado el martes, 5 de diciembre de 2023, 22:50


Estado Finalizado
Finalizado en martes, 5 de diciembre de 2023, 22:59
Tiempo empleado 9 minutos 27 segundos
Calificación 15,00 de 20,00 (75%)

Pregunta 1

Correcta

Se puntúa 5,00 sobre 5,00

Paciente que ingresa por tomar bebida con insecticida, se le mide la PRESION ARTERIAL. 70/40 HAY SIALO
ademas satura 80% , y tiene glasgow 8 - que farmaco usaria - que tratamiento inicia.

a. atropina - IOT

b. atropina - rcp avanzado 

c. ADRENALINA - atropina

d. atropina - rcp basico

e. ADRENALINA- intubacion

Respuesta correcta
La respuesta correcta es:
atropina - rcp avanzado

https://www.aulauss.edu.pe/mod/quiz/review.php?attempt=3057640&cmid=4478485 1/3
5/12/23, 23:01 post TEST . IOF - STATUS CONVULSIVO.: Revisión del intento

Pregunta 2

Correcta

Se puntúa 5,00 sobre 5,00

neonato del 8 meses de edad que ingresa ala emergencia con fiebre 38° c, ingresa con convulsion que dura
de convulsion por el tiempo es.....................que farmaco usaria ud. ...........que via usaria ........................

a. estacionario - diazepan - endovenosa

b. inicial - diazepan . venosa

c. refractario - diazepan - rectal

d. inicial- diazepan - rectal 

e. inicial - bromazepan . oral

Respuesta correcta
La respuesta correcta es:
inicial- diazepan - rectal

Pregunta 3

Correcta

Se puntúa 5,00 sobre 5,00

son signos de atropinizacion : es correcto.

a. sialorrea

b. midriasis 

c. defecacion del paciente

d. fasciculaciones musculares

e. miccion del paciente

Respuesta correcta

La respuesta correcta es:


midriasis

https://www.aulauss.edu.pe/mod/quiz/review.php?attempt=3057640&cmid=4478485 2/3
5/12/23, 23:01 post TEST . IOF - STATUS CONVULSIVO.: Revisión del intento

Pregunta 4

Incorrecta

Se puntúa 0,00 sobre 5,00

paciente de 36 años de edad que ingresa por quemadura en la parte anterior del miembro inferior , ademas de q
mienbro superior derecho en su cara anterior y posterior que compromete brazo, antebrazo y mano. qiue % de
comprometida

a. 27% 

b. 18%

c. 9%

d. 12%

e. 19%

Respuesta incorrecta.
La respuesta correcta es:
18%

https://www.aulauss.edu.pe/mod/quiz/review.php?attempt=3057640&cmid=4478485 3/3
5/12/23, 23:01 post TEST . IOF - STATUS CONVULSIVO.: Revisión del intento

Comenzado el martes, 5 de diciembre de 2023, 22:50


Estado Finalizado
Finalizado en martes, 5 de diciembre de 2023, 22:59
Tiempo empleado 9 minutos 27 segundos
Calificación 15,00 de 20,00 (75%)

Pregunta 1

Correcta

Se puntúa 5,00 sobre 5,00

Paciente que ingresa por tomar bebida con insecticida, se le mide la PRESION ARTERIAL. 70/40 HAY SIALO
ademas satura 80% , y tiene glasgow 8 - que farmaco usaria - que tratamiento inicia.

a. atropina - IOT

b. atropina - rcp avanzado 

c. ADRENALINA - atropina

d. atropina - rcp basico

e. ADRENALINA- intubacion

Respuesta correcta
La respuesta correcta es:
atropina - rcp avanzado

https://www.aulauss.edu.pe/mod/quiz/review.php?attempt=3057640&cmid=4478485 1/3
5/12/23, 23:01 post TEST . IOF - STATUS CONVULSIVO.: Revisión del intento

Pregunta 2

Correcta

Se puntúa 5,00 sobre 5,00

neonato del 8 meses de edad que ingresa ala emergencia con fiebre 38° c, ingresa con convulsion que dura
de convulsion por el tiempo es.....................que farmaco usaria ud. ...........que via usaria ........................

a. estacionario - diazepan - endovenosa

b. inicial - diazepan . venosa

c. refractario - diazepan - rectal

d. inicial- diazepan - rectal 

e. inicial - bromazepan . oral

Respuesta correcta
La respuesta correcta es:
inicial- diazepan - rectal

Pregunta 3

Correcta

Se puntúa 5,00 sobre 5,00

son signos de atropinizacion : es correcto.

a. sialorrea

b. midriasis 

c. defecacion del paciente

d. fasciculaciones musculares

e. miccion del paciente

Respuesta correcta

La respuesta correcta es:


midriasis

https://www.aulauss.edu.pe/mod/quiz/review.php?attempt=3057640&cmid=4478485 2/3
5/12/23, 23:01 post TEST . IOF - STATUS CONVULSIVO.: Revisión del intento

Pregunta 4

Incorrecta

Se puntúa 0,00 sobre 5,00

paciente de 36 años de edad que ingresa por quemadura en la parte anterior del miembro inferior , ademas de q
mienbro superior derecho en su cara anterior y posterior que compromete brazo, antebrazo y mano. qiue % de
comprometida

a. 27% 

b. 18%

c. 9%

d. 12%

e. 19%

Respuesta incorrecta.
La respuesta correcta es:
18%

https://www.aulauss.edu.pe/mod/quiz/review.php?attempt=3057640&cmid=4478485 3/3
12/12/23, 21:02 POST TEST SEMANA 15.: Revisión del intento

Comenzado el martes, 12 de diciembre de 2023, 21:00


Estado Finalizado
Finalizado en martes, 12 de diciembre de 2023, 21:02
Tiempo empleado 2 minutos 38 segundos
Calificación 20,00 de 20,00 (100%)

Pregunta 1

Correcta

Se puntúa 5,00 sobre 5,00

CAUSA mas frecuente de acv.

a. TODAS

b. AIT

c. isquemico 

d. hemorragico

e. HSA

Respuesta correcta

La respuesta correcta es:


isquemico

Pregunta 2

Correcta

Se puntúa 5,00 sobre 5,00

Paciente con ac de HTA ingresa por la cefalea mas intensa de su vida, se le realiza UNA TAC SE observa una lesion tipo fisher 4.
cual es su manejo.

a. ninguno

b. medico

c. solo quirurgico 

d. medico y quirurgico

e. solo medico

Respuesta correcta

La respuesta correcta es:


solo quirurgico

https://www.aulauss.edu.pe/mod/quiz/review.php?attempt=3061162&cmid=4479926 1/2
12/12/23, 21:02 POST TEST SEMANA 15.: Revisión del intento

Pregunta 3

Correcta

Se puntúa 5,00 sobre 5,00

Paciente ingresa con trastorno del sensorio ingresa y se le haca una TAC. SE OBSERVA coagulo lozalizado de 5 x 3 mm en fosa
posterior. QUE FISHER SERIA.

a. TRES 

b. DOS

c. CINCO

d. CUATRO

e. UNO

Respuesta correcta
La respuesta correcta es:
TRES

Pregunta 4

Correcta

Se puntúa 5,00 sobre 5,00

PACIENTE con glasgow 8, esta con trastorno del sensorio tiene una TAC DONDE SE OBSERVA lesiones biconvexa.

a. epidural 

b. INTRAPARENQUIMAL

c. HSA

d. subdural

e. SUBARACNOIDEO

Respuesta correcta
La respuesta correcta es:
epidural

https://www.aulauss.edu.pe/mod/quiz/review.php?attempt=3061162&cmid=4479926 2/2
12/12/23, 20:23 POST TEST URGENCIAS - PANCREATITIS - HDA. AULA A: Revisión del intento

Comenzado el sábado, 11 de noviembre de 2023, 20:50


Estado Finalizado
Finalizado en sábado, 11 de noviembre de 2023, 20:59
Tiempo empleado 9 minutos 23 segundos
Calificación 12,50 de 20,00 (62,5%)

Pregunta 1

Correcta

Se puntúa 5,00 sobre 5,00

Causa mas frecuente de hemorragia digestiva alta .

a. ulcera yeyuno

b. ulcera peptica 

c. ulcera ileon

d. esofagico

e. ulcera duodenal

Respuesta correcta

La respuesta correcta es:


ulcera peptica

Pregunta 2

Parcialmente correcta

Se puntúa 2,50 sobre 5,00

el paciente con pancreatitis segun ramzon alas 48 horas es falso afirmar

a. se evalua ascenso del hematocrito

b. se evalua ascenso del CO2 en sangre 

c. se evalua BUN

d. se evalua calcio serico

e. se evalua secuestro de liquidos mas de 4 litros

Respuesta parcialmente correcta.

La respuesta correcta es:


se evalua ascenso del hematocrito

https://www.aulauss.edu.pe/mod/quiz/review.php?attempt=3051722&cmid=4471723 1/2
12/12/23, 20:23 POST TEST URGENCIAS - PANCREATITIS - HDA. AULA A: Revisión del intento

Pregunta 3

Incorrecta

Se puntúa 0,00 sobre 5,00

Paciente mujer de 45 años de edad presenta hace 7 dias dolor abdominal mas ictericia.

ademas se hace una ecografia: HAY DILATACION del coledoco 12mm , ingresa con nauseas, vomitos no tolera el acostarse ni el
ingreso de los alimentos.

ES FALSO AFIRMAR en el tratamiento del paciente.

a. hidratacion endovenosa agresiva

b. pasa a colangioresonamcia abdominal para ver via biliar.

c. ingresar a cirugia colecistectomia 48 horas despues de calmar el dolor.

d. se hidrata, calma el dolor , baja la ictericia y pasa a cirugia

e. se hidrata , pasa a calmar el dolor luego a CPRE. 

Respuesta incorrecta.

La respuesta correcta es:


ingresar a cirugia colecistectomia 48 horas despues de calmar el dolor.

Pregunta 4

Correcta

Se puntúa 5,00 sobre 5,00

Paciente con dolor abdominal con presencia de un pancreas con 1 coleccion peripancreatica en la tomografia - cual es el grado de
severidad segun ATLANTA .------------- Y SEGUN BALTAZAR ES ------------------------------

a. leve - D 

b. moderado - B

c. leve - A

d. severa - D

e. severa - B

Respuesta correcta

La respuesta correcta es:


leve - D

https://www.aulauss.edu.pe/mod/quiz/review.php?attempt=3051722&cmid=4471723 2/2
TERCER EXAMEN TIPO ENAM

SEMINARIOS AVANZADOS 8. Varón de 30 años de edad, con múltiples parejas


sexuales. Acude con 10 días de enfermedad,
NOMBRE ……………………………….AULA………….. presentando una úlcera única en glande, limpia,
indolora y de bordes elevados. ¿Cuál es el
diagnóstico más probable?
A. Sífilis
INFECTOLOGIA
B. Chancroide
C. Herpes genital
1. ¿Qué diagnóstico consideraría para el siguiente
D. Linfogranuloma venéreo
cuadro clínico?: Fiebre, escalofríos, trastorno mental, E. Granuloma inguinal
artralgias, mialgias, anemia severa, lesiones cutáneas
purpúreas: 9. Un agricultor de 25 años de edad ingresa a
A. Septicemia Emergencia con diaforesis profusa, dolor abdominal
B. Sepsis por S. aureus intenso, edema facial periorbitario severo y mialgias
C. Brucelosis severas. Informa que fue sido mórdido por un "animal
D. Malaria ponzoñoso" mientras trabajaba en el campo. ¿A qué
E. Bartonelosis cuadro clínico corresponde?
A. Lutzomia
2. Vector de la bartonelosis: B. Loxocelismo
A. Aedes C. Latrodectismo
B. Anopheles D. Leptospirosis
C. Lutzomia E. Flevotomus verrucaru
D. Leishmania
E. Bartonella
10. Varón de 40 años que presenta en el brazo derecho
3. En el Perú, la brucelosis es fundamentalmente
lesión máculopapulosa de 3 cm, pruriginosa, indolora
producida por:
A. Brucella bovis que progresa a vesícula y luego a costra negruzca
B. Brucella abortus rodeada de edema. Ocupación crianza de ganado
C. Brucella melitensis lanar y vacuno. Niega fiebre. Examen: Pulso: 115 x
D. Brucella canis minuto, FR: 24 x minuto, PA: 100/60 mmHg. ¿Cuál es
E. Brucella suis la presunción diagnóstica?
A. Lesión por Pseudomona
4. Paciente que ingresó de la selva, presentó a las B. Loxocelismo
pocas horas malestar general, anorexia, dolor C. Lactrodactismo
abdominal y diarreas acuosas abundantes. Al día D. Leishmaniasis
siguiente se agrega debilidad, vómitos hipotensión; E. Carbunco
en el cultivo de heces se aisló Vibrio cholerae. ¿Qué
tipo de receptor de membrana está comprometido en NEUROLOGIA
el desarrollo de esta enfermedad?:
A. Gangliósido Gm1 11. Mujer de 32 años, llega a consulta con cefalea
B. Gangliósido Gm2 intensa, vómitos y rigidez de nuca. El LCR presenta
C. Gangliósido Gm3 pleocitosis linfocitaria. ¿Cuál es el diagnóstico más
D. Proteína G probable?
E. Receptor PDGF (factor de crecimiento a. Meningitis viral
derivado de plaquetas) b. Meningitis por hongos
5. ¿Cuál de los siguientes medicamentos se asocia a c. Meningitis bacteriana
colitis por clostridium difficile? d. Meningitis por VIH
A. Ciprofloxacino e. Hemorragia subaracnoidea
B. Levofloxacino 12. Mujer con fiebre, signos meníngeos sin lesiones
C. Clindamicina focales ni signos de hipertensión endocraneana.
D. Metronidazol ¿Cuál es el paso inmediato a seguir?
E. Vancomicina a. Realizar una RM cerebral
6. Paciente de 22 años, con tiempo de enfermedad de 3 b. Realizar una TC cerebral
días. Examen ginecológico: vesículas en racimo c. Solicitar hemocultivos
sobre labio mayor y adenopatías homolaterales d. Realizar fondo de ojo
dolorosas. ¿Cuál es el diagnóstico probable – e. Realizar punción lumbar
Tratamiento ? 13. Dicha punción lumbar se realiza entre: -
A. Chancro sifilítico – peniclina a. L1 – L2 B. L2- L3
B. Condiloma vulvar - podofilina c. L3 - L4 D. L4 - L e. L5 - S1
C. Herpes genital - Aciclovir
D. Absceso perineal - clindamicina
E. Linfogranuloma venéreo - azitromicina 14. Una madre trae a emergencia a su niño de 5 años,
quien presenta sialorrea, diarrea y debilidad muscular.
Al examen: se encuentra taquicardia y
7. Varón de 30 años de edad, acude por úlcera genital fasciculaciones. Tórax: roncantes, sibilantes y
en glande, de 10 días de evolución, con antecedente subcrepitantes en ambos campos pulmonares.
de múltiples parejas. Tiene VDRL reactivo de 2 Pupilas mióticas casi puntiforme.
diluciones. ¿Cuál es el examen que confirma el ¿Dentro del manejo inicial de este niño cuál es el
diagnóstico? fármaco que indicaría?
A. Nuevo VDRL a. Sulfato de atropina
B. FTA-ABS b. Fisostigmina
C. RPR c. Pilocarpina
D. Ig G para herpes d. N-acetil cisteína
E. Ig M para herpes e. Flumazenilo.
D. Bicarbonato menos de 15 mEq/L
E. Cuerpos cetónicos en orina
15. En cuál de las siguientes patologías se agrega
corticoides al tratamiento específico?
a. Meningitis tuberculosa 22. Paciente de 45 años que presenta glicemia en ayunas
b. Tuberculosis miliar de 118 y 135 mg / 100 ml, tomados en días diferentes.
c. Tuberculosis primaria ¿Cuál sería la conducta a seguir?
d. Pleuritis tuberculosa A. Iniciar tratamiento con Insulina NPH
e. Tuberculosis peritoneal B. Repetir el examen en un mes
C. Iniciar Hipoglicemiantes orales
16. En la enfermedad cerebro vascular. ¿Cuál es el D. Solicitar hemoglobina glicosilada
examen que permite diferenciar el ictus isquémico del E. Solicitar tolerancia oral de glucosa
hemorrágico?
a. Tomografía computarizada sin contraste
b. Tomografía computarizada con contraste 23. Son intolerantes a la glucosa:
c. Estudio de LCR a. Glicemia basal de 115
d. Electroencefalograma b. Glicemia a los 120 minutos de TTOG entre
e. Doppler transcraneal de pulso 140 y 200.
c. Basal de 140
17. Paciente mujer de 45 años realiza una fuerte d. Glicemia a los 120 minutos de TTOG entre
discusión con su pareja y decide tomar un frasco de 200 y 202.
20 pastillas de diazepan de 0.5mg llega al hospital en e. Curva de glicemia patológica tras estimulo
mal estado general con trastorno del sensorio , cual con GC.
antídoto usara usted.
a. Pilocarpina
b. Flumazenilo 24. En el tratamiento de Diabetes Mellitus de tipo II ¿Cuál
c. Acetilsisteina de los siguientes fármacos
d. Naloxona disminuye la producción hepática de glucosa y produce pérdida
e. Protamina de peso?
A. Pioglitazona
18. Paciente varon de 75 años ingresa ala emergencia B. Glimepirida
por petequias , manchas color verde marron en la piel C. Metformina
del tronco y extremidades , tiene como antecedentes D. Clorpropamida
tomas Warfarina 5 mg una tableta cada 24 horas de E. Glibenclamida
lunes a viernes, por antecedente de fibrilación
auricular, lo alarma porque hace 30min observa 25. La mejor prueba para inferir la efectividad del control
sangrado en la orina, que antídoto usara ud : de la Diabetes Mellitus es el dosaje de:
a. Protamina A. Glucosa en orina
b. Vitamina C B. Glicemia post prandial
c. Vitamina K C. Glicemia en ayunas
d. Naloxona D. Curva de tolerancia a la glucos
e. Oxigeno E. Hemoglobina glicosilada.

26. Paciente varón de 45 años obeso, ingresa a la


19. Para un mejor control de la coagulación que exámenes emergencia con signos graves de deshidratación , se
de sangre solicitaría ud para ver el estado de le realiza exámenes de sangre básicos:
coagulación del paciente: Glicemia: 800 urea: 45 creatinina . 1.5
a. Tiempo de protrombina orina: presencia de cuerpos cetónicos
b. Tiempo de tromboplastina Sodio sérico: 155 Potasio: 6.0
c. Tiempo de coagulación Según dicho cuadro: calcule la osmolaridad
d. Tiempo de sangría efectiva:………………..
e. Recuento de plaquetas Diagnostico mas probable.
20. Paciente varón de 45 años con antecedentes de a. 314.4 - CAD
alcoholismo, ingiere una botella de anisado de b. 324.4 - SHH
procedencia dudosa, a los 4 horas es llevado al c. 334.4 - CAD
hospital en malas condiciones de salud, presenta d. 354.4 - MIXTO ( CAD + SHH)
respiración taquipneica y los siguientes signos vitales: e. 354.4 - SHH
PA: 70 /50 FC : 125 FR: 30
AGA Y ELECTROLITOS: Ph. 7:0 Pa CO2: 20
HCO3 . 10 27. Mujer obesa de 115 kg talla: 1,65 cm , ingresa por
Presenta el siguiente diagnostico con trastorno acido dolor abdominal tipo cólico en HCD irradiado a la
base: espalda post ingesta de comida copiosa y grasosa.
a. Intoxicación por etanol - alcalosis respiratoria Ingresa con los siguientes exámenes de sangre:
b. Intoxicación por metanol- alcalosis respiratoria Glucosa: 255 urea: 55 creatinina : 2.0 orina:
c. Intoxicación por etanol - acidosis metabolica cuerpos cetónicos y piuria
d. Intoxicación por benzodiacepinas – acidosis Signos vitales:
metabolica PA: 70/50 FC: 110 FR: 28
e. Intoxicación por metanol – acidosis metabolica AGA Y ELECTROLITOS: Sodio : 148 Potasio:
5.0 HCO3. 20 PaCO2. 15
ENDOCRINOLOGIA DIAGNOSTICO:
a. Shock séptico – CAD
b. Shock cardiogénico_ SHH
21. Uno de los siguientes hallazgos NO es de c. Sepsis foco urinario - CAD
presentación necesaria en la cetoacidosis diabética: d. SEPSIS - SHH
A. Acidosis metabólica e. Sepsis – CAD
B. Glicemia mayor de 400mg/dL
C. Hiperpotasemia
28. ENAM 2005-09: Mujer de 38 años de edad, consulta E. Ectasia vascular.
por incremento de peso. Su glicemia en ayunas es de
112 mg/dL y postprandial a las dos horas de 150 35. Cuál es la causa más frecuente de muerte debida a
mg/dL. Índice de masa corporal: 33. El tratamiento hemorragia de origen gastrointestinal?:
recomendado es: A. Rotura de varices esofágicas
A. Biguanida B. Ulcera gástrica sangrante
B. Insulina de depósito C. Diverticulos en el colon
C. Sulfonilurea de acción corta D. Ulcera duodenal sangrante
D. Sulfonilurea de acción prolongado E. Polipos en el colon
E. Dieta y bajar de peso

36. Un varón de 60 años de edad previamente sano


acude a consulta por hemorragia rectal masiva. ¿Cuál
29. Mujer de 38 años, desde hace 10 meses refiere es el diagnóstico más frecuente?
nerviosismo, intolerancia al calor, cansancio, labilidad A. Enfermedad diverticular del colon
emocional y pérdida de peso. Al examen físico: B. Colitis ulcerosa
disminución de peso, habla rápido, exoftalmos C. Hemorroides externas
evidente. Cuello: a la palpación tiroides con aumento D. Colitis isquémica
de volumen en forma difusa. Corazón: ruídos E. Carcinoma del colon
cardíacos taquicárdicos. ¿Cuál es el diagnóstico
probable? 37. Paciente obesa hace 3 dias presenta dolor abdominal
A. Tiroiditis de Hashimoto tipo colico , en el hipocondrio derecho mas vomitos
B. Enfermedad de Graves Basedow de contenido biliar postprandial además hay fiebre
C. Tiroiditis subaguda hoy dia,
D. Enfermedad de Plummer
Al examen presenta Murphy + , en la ecografía
presenta calculo incrustado en el conducto cístico.
Diagnostico mas probable.
30. Paciente varón de 35 años es traído a Emergencia con
trastorno del sensorio, obnubilado, polipneico, se a. Colelitiasis
evidencia mucosas orales secas. Estudio de Glicemia b. Colico biliar
880 mg/dL, pH =7.20, HCO3=13, pCO2 =24, cetonuria c. Colecistitis aguda
(++), Na+ sérico:145. ¿Cuál es el manejo ideal? d. Calculo en el coducto común
A. Hidratación con agua destilada, insulina EV. e. Cirrosis biliar secundaria
B. Hidratación enérgica con suero 38. Diarrea mas común en el adulto puede cursar con
fisiológico, insulina en bolo y en infusión. sangre en las heces esta asociado con síndrome de
C. Hidratación con suero hipotónico, reiter , y síndrome de Guiliam barre. Agente
bicarbonato de sodio EV. etiologico.
D. Insulina EV en bolo y bicarbonato de sodio.
a. Shiguella
b. Salmonella
GASTROENTEROLOGIA
c. Campilobacter
d. Stafilococos
31. El diagnóstico de hepatitis viral B aguda se realiza
por: e. Clostridium dificille
A. Presencia de antígeno de superficie
B. Detección del DNA HBV por reacción de cadena
de polimerasa 39. De los siguientes enunciados. ¿Cuál se relaciona con
C. Detección del RNA HVB por eacción de cadena de cuadro de colestasis?
polimerasa
D. Desaparición del anti-HBc A. Elevación de bilirrubinas a predominio
E. Elevación del anti-HBc indirecto
B. Valores normales de FA y GGT
32. La efectividad de la vacuna contra la hepatitis B se C. Elevación marcada de AST y ALT
mide con el dosaje de: D. Elevación de bilirrubinas predominio directo,
A. Anti-HBc FA y GGT
B. Hbc Ag
C. Anti-HBe
D. Anti-HBs 40. Son factores de riesgo para cáncer gástrico, excepto:
E. HBs Ag A. Postgastrectomía por úlcera péptica
33. Paciente varón de 60 años cursa con diarrea crónica B. Anemia perniciosa
desde hace mas de 6 meses , en los últimos 7 dias C. Helicobacter Pylori
presenta heces con sangre , hoy amanece muy D. Pólipo glandular fúndico
debilitado se le realiza una endoscopia alta y baja :
encuentran granulomas en esofago, colon, con
NEUMOLOGIA
presencia de algunas fistulas. Diagnostico mas
probable.
41. En un paciente con diagnóstico de EPOC, la
a. Síndrome intestino irritable espirometría muestra:
b. Colitis ulcerativa A. Disminución de VEF1 y CVF
c. Enfermedad de Crohn B. Disminución del VEF1 y aumento de CVF
d. Linfoma gastrico C. Disminución de CFV y VEF1 normal
e. Cancer gastrico D. Aumento del VEF1 y disminución de VR
34. La causa más frecuente de hemorragia digestiva baja
es: 42. Paciente varón de 70 años, es hospitalizada en UCI
A. Diverticulosis por intoxicación por benzodiazepinas se encuentra en
B. Malformación arteriovenosa destete de ventilación mecánica. Al cuarto día
C. Hemorroides presenta cambio de las características e incremento
D. Angiodisplasia de secreciones bronquiales, asociada a fiebre y shock.
En la radiografía de tórax: infiltrado alveolar difuso. El
tratamiento antibiótico empírico ideal es: CARDIOLOGIA
A. Amikacina-ceftazidima-ciprofloxacina
B. Ceftriaxona-azitromicina-amikacina
C. Meropenem-vancomicina-amikacina
D. Ceftazidima-oxacilina-amikacina 51. Varón de 55 años que presenta compromiso de
43. Mujer de 82 años con fractura de cadera y postrada, conciencia, dolor torácico y
presenta disnea súbita e hipoxemia. FC=102 lat/min', T dificultad respiratoria. Examen: Ingurgitación yugular (++),
37°C. Si sospecha de TEP. ¿Cuál sería su manejo taquicardia con ruidos cardiacos
inicial? débiles y presión diferencial menor de 30 mm Hg e hipotensión
A. Trombolíticos arterial. ¿Cuál es el diagnostico probable?
B. Heparina EV A. Infarto pulmonar
C. Heparina SC B. Aneurisma de aorta torácica
D. Warfarina C. Neumotórax
44. La Enfermedad Diarreica Aguda caracterizada por D. Taponamiento cardiaco agudo
transporte activo de solutos a la luz intestinal, es la E. Enfermedad de Ebstein descompensada
diarrea: 52. Marque lo incorrecto en el dolor por pericarditis:
A. Por invasión de la mucosa A. Se produce por inflamación o infiltración del
B. Osmótica pericardio parietal
C. Secretora B. Puede identificarse un frote
D. Por aumento de la motilidad C. Puede irradiarse a los hombros, cuello y
E. Por mala absorció, región dorsal
45. Señale la definición correcta de diarrea aguda: D. Mejora con el decúbito y se incrementa al
A. Tiempo de duración es inferior a 14 días sentarse
B. Tiempo de duración es inferior a 7 días E. Puede causar taponamiento cardíaco
C. Tiempo de duración es inferior a 5 días 53. ¿Cuál de las siguientes pruebas da diagnóstico
D. Tiempo de duración es inferior a 10 días preciso de pericarditis?
E. Tiempo de duración es inferior a 21 días A. Punción pericárdica
B. Electrocardiograma
46. Criterio de pronóstico al Ingreso en pancreatitis C. Radiografía de tórax
aguda: D. Ecocardiograma
a. TGO 400 E. Tomografía
b. Pa O2 50 mmHg 54. Paciente que luego de sufrir agresión con arma
c. Deficit de bases blanca a nivel paraesternal izquierdo
d. Secuestro liquidos presenta hipotensión, ruidos cardíacos poco audibles y gran
e. Hipocalcemia ingurgitación yugular. ¿Cuál
es diagnóstico más adecuado?:
A. Edema agudo de pulmón
47. ENAM 2007-12: En la pancreatitis aguda del adulto, B. Infarto agudo de miocardio
cuál de los siguientes criterios no corresponde a los C. Hemopericardio
de Ranson? D. Tromboembolismo pulmonar
A. Leucocitosis >16000/𝑚𝑚3 E. Neumotórax
B. Edad <55 años 55. Paciente con disnea y Ortopnea . En el examen físico
C. Hiperglicemia >200 mg/dL se encuentra pulso paradójico ¿Cuál es el
D. LDH en suero >400 UI/L diagnóstico más probable?
E. AST en suero >250 UI/L a. PERICARDITIS CONSTRICTIVA
b. MIOCARDIOPATIA DILATADA
c. TROMBOSIS VENOSA PROFUNDA
48. SSALUD 2012 Un paciente de 42 años ingreso hace 6 d. MIOCARDITIS AGUDA
días con un cuadro de pancreatitis aguda de origen e. IMA
biliar- la mala evolución obliga a determinar si
presenta necrosis pancreática. ¿Qué prueba 56 Paciente que sufre trauma torácico severo y presenta
diagnóstica solicitaría? hipotensión, elevación de la presión venosa central con pulso
A. Ecografia abdominal con contraste endovenoso filiforme. ¿Cuál es el diagnóstico más probable?
B. Colangio-resonancia magnética A. Hemotórax
C. Radiografia simple de abdomen B. Neumotórax
D. Colangiopancreatografia retrograda endoscópica C. Taponamiento cardiaco
E. Tomografia computarizada con contraste D. Fracturas costales
Endovenoso E. Neumomediastino

49. Cuál es la causa más frecuente de hemorragia 57. El taponamiento cardiaco traumático se produce
masiva?: cuando el hemopericardio acumula
A. Diverticulosis a. 250 ml de sangre
B. Diverticulitis b. 500 ml de sangre
C. Cáncer de colon c. 1000 ml de sangre
D. Pólipos d. 1500 ml de sangre
E. Angiodisplasia e. 2000 ml de sangre
50. Paciente con hemorragia digestiva alta por ulcera 58. Mujer de 60 años de edad, consulta por edema de
duodenal se decide tratamiento qx, cual es la mejor
miembro inferior izquierdo.Se hace diagnóstico de
conducta:
trombosis venosa profunda, El edema es producido
A. Parche de la lesión sangrante
por:
B. Antrectomia +vagotomiatroncular
A. Disminución de la resistencia periférica
C. Vagotomia selectiva maspiloroplastia
B. Disminución de las proteínas plasmáticas
D. Vagotomiaultraselectiva
C. Aumento de la permeabilidad capilar
E. Reseccion parcial del estomago + billrot II
D. Obstrucción de drenaje linfático
E. Aumento de la presión capilar 65. Joven de 26 años ingresa con lesiones bilateral en la
59. En un paciente operado de cadera, obeso, con región inguinal tipo placas con bordes circinados
aparición súbita de disnea, dolor torácico, tos, esputo macerada , muy pruriginosas diagnostico.
hemoptoico; se debe sospechar: a. Tiña corporis
A. Pericarditis aguda b. Tiña cruris
B. Aneurisma disecante c. Dermatitis fúngica
C. Pleuresía con derrame
d. Dermatitis de contacto
D. Neumonía lobar
e. Psoriasis invertida
E. Embolismo pulmonar

60. Paciente con pulso arrítmico y deficitario. ¿A qué 66. Paciente mujer con antecedente de varices en
enfermedad corresponde?
miembros inferiores bilateral ingresa por lesión
A. Taquicardia supraventricular
unilateral tipo ulceras en miembro inferior derecha con
B. Fibrilación auricular
C. Taquicardia ventricular signos de flogosis local, eritematosa , y pruriginosa
D. Fibrilación ventricular de mas o menos 10 cm de diámetro diagnostico.
E. Bloqueo AV a. Inpetigo
b. Varicela
c. Celulitis
DERMATOLOGIA d. Dermatofitosis
e. Psoriasis
61. Paciente obeso de 45 años con antecedente de 67. Paciente varon de 65 años hace dos días presenta
diabetes mellitus inicia cuadro de lesiones tipo placas insolación por 4 horas, desde ayer presenta dolor en
en cuero cabelludo que descaman asi mismo en codos hemitórax derecha a nivel de la tetilla derecha y hoy
bilateral asi como en rodillas región anterior. amanece con lesiones multiples confluentes en
Diagnostico hemitórax derecho que le produce dolor y ardor local
a. Impétigo con prurito intenso. Diagnostico.
b. Celulitis a. Psoriasis
c. Dermatitis seborreica b. Dermatitis de contacto
d. Psoriasis vulgar c. Herpes zoster
e. Eritrodermia psoriasica d. Dermatitis atópica
e. Dermatofitosis
68. Tratamiento de elección en la dermatitis por herpes
62. Paciente adulto mayor de 75 años ingresa por 2 dias virus.
antes de ingreso fiebre cuantificada 39° C dos días a. Aciclovir - mupirocina
después observa lesiones tipo ampollas en hemicara b. Aciclovir – prednisona
derecha unilateral que abarca región ocular y de c. Prednisona – vitamina A
pabellón auricular homolateral que produce dolor d. Prednisona - eritromicina
antes de Salir, asi como intenso prurito,además se e. Eritromicina – Aciclovir.
observa compromiso de paralisis del siguiente par
craneal ………………………………………….. 69. Lesión en pene tipo arrosariado como coliflor crónica
diagnostico. multiples que compromete cuerpo peneano y surco
a. Varicela- VII balanoprepucial . diagnostico – agente etiológico.
b. Herpes tipo 2 - V a. Herpes virus – virus tipo 2.
c. Sindrome de ramsay Hunt - V b. Linfogranuloma venéreo – bacterias gram - .
d. Sindrome da ramsay Hunt - VII c. Sífilis - chancroide
e. Dermatitis de contacto - III d. Verruga viral – papiloma virus
e. Micosis – candida

63. Mujer de 24 años con varias parejas de contacto


sexual ingresa porque 5 dias antes de ingreso 70. Paciente con lesiones en placas en zonas como el
presenta secreción vaginal abundante tipo leche cuello, codos , además en zonas de rodillas
cortada que se pega alas paredes de la vagina, que acompañadas con escamas , hacemos el raspado de
se desprenden fácilmente. Diagnostico. las mismas con una lamina portaobjeto y se aprecia
a. Infección por tricomona el desprendimiento de …………………………………
b. Infección por candida al fondo deja puntos sangrantes conocido como
c. Infección por herpes virus …………………………….patognomonico
d. Infección por ricketsia de………………………..
e. Infección por gram negativas a. Membrana de brush - signo de Duncan -
Urticaria
b. Membrana de Duncan - signo de austpitz –
Urticaria crónica
64. Paciente obesa que ingresa por lesiones tipo placas c. Membrana de Duncan - signo de auspitz -
en zonas de pliegue como axilas, ingles debajo de la Psoriasis vulgar
mama bilateral , muy pruriginosas y eritematosas con d. Membrana de auspitz - signo de duclan Bucley -
presencia de satelitosis. Diagnostico. Dermatitis atópica
a. Intertrigo e. Membrana de Duncan - signos de auspitz -
b. Psoriasis Penfigo vulgar.
c. Impetigo
d. Celulitis NEFRO . PSIQUIATRIA.
e. Dermatitis atópica.
71. Madre con TBC pulmonar acude con su hijo
hipotónico e hipoactivo. Refiere que solo lo alimenta
con agua, infusiones y sopas, por falta de dinero. c. Síndrome confusional agudo
¿Cuál es el trastorno hidroelectrolítico que presenta? d. Demencia frontal
A. Hiperfosfatemia e. Depresión
B. Hipernatremia
C. Hiponatremia
D. Hiperkalemia
E. Hipokalemia 79. Los principales síntomas del trastorno por crisis de angustia
son:
a. Depresión y despersonalización.
b. Sentirse confundido y tener dificultades de concentración.
72. Varón de 92 años, acude a EMG por episodio c. Un miedo extremo y una sensación de muerte inminente.
convulsivo. Examen físico: hemodinamicamente d. Ansiedad anticipatoria y fobia social.
estable, no edemas. Laboratorio. Sodio sérico en 108 e. Alteraciones mesicas y preocupación de que episodio se
mEq/L. En el tratamiento, la reposición de sodio con repita
solución salina es al :
A. 5% B. 3% C. 9%
D. 10% E. 15% 80. Varón de 45 años de edad con conflictos conyugales
desde hace 2 años. Se queja de dificultad para
conciliar el sueño, contractura muscular, dolor de
73. Hiponatremia 116mEq + Na en orina >20mEq + nuca y espalda, irritabilidad, parestesias en miembros
Edema en MMII: superiores, expectación aprehensiva. El diagnóstico
a. IRC probable es:
b. ICC a. Hipocondriasis
c. Cirrhosis hepatica b. Trastorno conversivo
d. SIHAD c. Trastorno de adaptación
e. Diabetes insipida d. Dolor psicógeno
e. Trastorno de ansiedad generalizada

74. Paciente de 19 años de edad, presenta sed y poliuria


(4-5 L/día). El sedimento
urinario es normal, sin glucosuria ni proteinuria. La densidad ROBINSON LEON Z
urinaria es 1.001 y la natremia 137 mEq/L. DOCENTE USS
el diagnóstico más probable es:
A. Insuficiencia renal crónica
B. Diabetes insípida central
C. Diabetes insípida nefrogénica
D. Síndrome de secreción inapropiada de ADH
E. Polidipsia primaria

75. Mujer de 45 años de edad con sepsis a punto de


partida gastrointestinal. Se tiene los siguientes
resultados: pH 7,26, pO2 74 mmHg, pCO2 32mmHg,
HCO3 10 mEq/L, Na 128 MEq/L, K:3 mEq/L,
Cl 96 mEq/L. el diagnóstico gasométrico es acidosis
Metabólica…con anion gap….:
A. Compensada/alto
B. Descompensada/alto
C. Descompensada/normal
D. Compensada/normal
E. Descompensada/bajo.

76. Cuál es la manifestación más frecuente en la


depresión?
a. Suicidio
b. Bajo rendimiento
c. Alcoholismo
d. Pseudemencia
e. Agresividad

77. ¿En qué condición mental existe mayor riesgo de


suicidio?
a. Depresión
b. Ansiedad
c. Esquizofrenia
d. Oligofrenia
e. Trastorno de personalidad

78. Varón de 85 años de edad. Desde hace un mes


presenta pérdida de interés por lo que
lo rodea, olvidos frecuentes, insomnio, irritabilidad y pérdida de
peso. Tiene un test minimental normal. ¿Cuál es el diagnóstico
más probable?
a. Demencia vascular
b. Demencia tipo Alzheimer
21/11/23, 23:00 poste test INFECCIOSAS - DENGUE: Revisión del intento

Comenzado el martes, 21 de noviembre de 2023, 22:54


Estado Finalizado
Finalizado en martes, 21 de noviembre de 2023, 23:00
Tiempo empleado 5 minutos 52 segundos
Calificación 20,00 de 20,00 (100%)

Pregunta 1

Correcta

Se puntúa 5,00 sobre 5,00

No es signo de alarma de dengue.

a. alteración del sensorio

b. sangrado activo del paciente mucosas

c. dolor abdominal

d. elevacion del hematocrito

e. cefalea intensa 

Respuesta correcta
La respuesta correcta es:
cefalea intensa

Pregunta 2

Correcta

Se puntúa 5,00 sobre 5,00

cuantos serotipos de dengue hay en el mundo:

a. 2

b. 3

c. 5

d. 1

e. 4 

Respuesta correcta

La respuesta correcta es:


4

https://www.aulauss.edu.pe/mod/quiz/review.php?attempt=3054366&cmid=4473359 1/2
21/11/23, 23:00 poste test INFECCIOSAS - DENGUE: Revisión del intento

Pregunta 3

Correcta

Se puntúa 5,00 sobre 5,00

en el tratamiento del dengue con signos de alarma. debo considerar excepto

a. mantener una flujo urinario optimo

b. mantener diuresis mas de 0.5 ml/jg/ hora

c. vigilar descenso de plaquetas 

d. si no mejora, considerar que hay samgrado oculto.

e. tener buena perfusion de tejidos perifericos y organos perifericos.

Respuesta correcta
La respuesta correcta es:
vigilar descenso de plaquetas

Pregunta 4

Correcta

Se puntúa 5,00 sobre 5,00

la enfermedad de WEILL se le llama tanbien a:

a. leptospirosis moderada

b. leptospirosis severa y grave 

c. leptospirosis con complicaciones tempranas

d. leptospitosis leve

e. leptospirosis moderada grave

Respuesta correcta
La respuesta correcta es:
leptospirosis severa y grave

https://www.aulauss.edu.pe/mod/quiz/review.php?attempt=3054366&cmid=4473359 2/2
14/11/23, 23:11 POSTEST SEMANA 11.: Revisión del intento

Comenzado el martes, 14 de noviembre de 2023, 23:01


Estado Finalizado
Finalizado en martes, 14 de noviembre de 2023, 23:09
Tiempo empleado 8 minutos 51 segundos
Calificación 20,00 de 20,00 (100%)

Pregunta 1

Correcta

Se puntúa 5,00 sobre 5,00

CAUSA mas frecuente de hemorragia digestiva baja benigna:

a. cancer colorrectal

b. enfermedad diverticular 

c. hemorroides

d. fisura anal

e. fistula anal

Respuesta correcta

La respuesta correcta es:


enfermedad diverticular

Pregunta 2

Correcta

Se puntúa 5,00 sobre 5,00

El indice de schok en HDB resulta de :

a. presion sistolica/ frecuencia cardiaca

b. frecuencia cardiaca/ presion arterial diastolica

c. frecuencia cardiaca/ presion arterial sistolica 

d. frecuencia cardiaca/ presion arterial media

e. presion sistolica/ presion diastolica

Respuesta correcta

La respuesta correcta es:


frecuencia cardiaca/ presion arterial sistolica

https://www.aulauss.edu.pe/mod/quiz/review.php?attempt=3052807&cmid=4472200 1/2
14/11/23, 23:11 POSTEST SEMANA 11.: Revisión del intento

Pregunta 3

Correcta

Se puntúa 5,00 sobre 5,00

Paciente de 70 años procedente de la sierra de cajamarca, ingresa ala emergencia por dolor abdominal en fosa iliaca izquierda hace
2 dias, hace 1 dia de agreda de fiebre y mal estado general, hace 2 horas hace deposiciones rojo vinosas ( hematoquecia ) .
diagnostico mas probable.

a. hematoquezia

b. cancer de colon

c. angiodisplasia

d. enfermedad diverticular. 

e. ulcera peptica sangrante

Respuesta correcta

La respuesta correcta es:


enfermedad diverticular.

Pregunta 4

Correcta

Se puntúa 5,00 sobre 5,00

En un paciente varon de 45 años , ingresa por dolor abdominal hace 7 dias,


tiene ausencia de flatos y deposiciones hace 2 dias, al examen abdomen blando depresible

con ausencia de RHA, en la Radiografia de abdomen se ven niveles en signos de pilas de moneda. Obstrucción de que asa
..........................

a. colon sigmoidea

b. recto

c. intestino delgado 

d. colon ascendente

e. colon

Respuesta correcta
La respuesta correcta es:
intestino delgado

https://www.aulauss.edu.pe/mod/quiz/review.php?attempt=3052807&cmid=4472200 2/2

You might also like